Азот относительная атомная масса: Чему равна относительная молекулярная масса азота (N2)?

Содержание

Чему равна относительная молекулярная масса азота (N2)?

Помогите 1198 номер решить

Зменшення тертя внаслідок розташування твердих котків між по-верхнями, що ковзають одна поэ одній, добре відоме. Скористайтесьдодатковими джерелами ін … формації та підготуйте повідомленняпро відповідні історичні приклади.​​

яким є тиск газу в колбі В якщо тиск газу в колбі А дорівнює 100 гПа?​

Найди давление атмосферы в шахте на глубине 801 м, если на поверхности Земли давление нормальное (101300 Па). Ответ (округли до десятых): давление атм … осферы в шахте равно кПа. Пожалууйстаааа , срочно помогимтееее

Напруга на ділянці кола дорівнює 36 В. З якої висоти має впасти вантаж масою 72 кг, щоб сила тяжіння виконала таку саму роботу, яку виконує електричне … поле, переміщуючи ділянкою кола заряд 900 Кл?

Помогите решить: Воздушный шарик объемом 4дм³ имеет подъемную силу, определить вес шарика?

На рисунку зображено електричне коло, що складається з однакових резисторів опо-ром 10 Ом кожний.

Чому дорівнює опір між точками А та в?​

1.Чому дорівнює сила, яка виштовхує повністю занурене в рідину або газ тіло? 2.Запишіть формулу для визначення сили Архімеда? 3.Від чого залежить сила … Архімеда? 4.До шальок терезів підвішені дві однакові залізні кульки. Чи порушиться рівновага, якщо кульки опустити одну в воду, а іншу — у гас? Відповідь поясніть. 5.До шальок терезів підвішені дві гирі однакової ваги: фарфорова і залізна. Чи порушиться рівновага терезів, якщо гирі опустити в посудину з водою? Відповідь поясніть. 6.У яких випадках на тіло, занурене в рідину, не діє виштовхувальна сила? Чому? 7.Яке значення сили Архімеда, що діє на повністю занурений у воду мідний брусок масою 840г? 8.На скільки важча доросла людина з об’ємом тіла 0,07 м3 в повітрі, ніж у воді? 9.Скільки важить тіло об’ємом 30см3 у гасі, якщо його вага в повітрі 6 Н?

ПОМОГИТЕ ПОЖАЛУЙСТА!!!!

Приготовьтесь отправиться в тур по Лондону. Обсудите, какие достопримечательности вы хотели бы увидеть и почему. Составьте список достопримечательн … остей, которые стоит увидеть за один день. ​

ОТНОСИТЕЛЬНАЯ АТОМНАЯ МАССА | Энциклопедия Кругосвет

ОТНОСИТЕЛЬНАЯ АТОМНАЯ МАССА. Английский ученый Джон Дальтон (1766–1844) на своих лекциях демонстрировал студентам выточенные из дерева модели атомов, показывая, как они могут соединяться, образуя различные вещества. Когда одного из студентов спросили, что такое атомы, он ответил: «Атомы – это раскрашенные в разные цвета деревянные кубики, которые изобрел мистер Дальтон».

Конечно, Дальтон прославился не своими «кубиками» и даже не тем, что в двенадцатилетнем возрасте стал школьным учителем. С именем Дальтона связано возникновение современной атомистической теории. Впервые в истории науки он задумался о возможности измерения масс атомов и предложил для этого конкретные способы. Понятно, что непосредственно взвесить атомы невозможно. Дальтон рассуждал только о «соотношении весов мельчайших частиц газообразных и других тел», то есть об относительных их массах.

И поныне, хотя масса любого атома в точности известна, ее никогда не выражают в граммах, так как это исключительно неудобно. Например, масса атома урана – самого тяжелого из существующих на Земле элементов – составляет всего 3,952·10–22 г. Поэтому массу атомов выражают в относительных единицах, показывающих, во сколько раз масса атомов данного элемента больше массы атомов другого элемента, принятого в качестве стандарта. Фактически это и есть «соотношение весов» по Дальтону, т.е. относительная атомная масса.

В качестве единицы массы Дальтон принял массу атома водорода, а для нахождения масс других атомов он использовал найденные разными исследователями процентные составы различных соединений водорода с другими элементами. Так, по данным Лавуазье, в воде содержится 15% водорода и 85% кислорода. Отсюда Дальтон нашел относительную атомную массу кислорода – 5,67 (в предположении, что в воде на один атом водорода приходится один атом кислорода). По данным английского химика Уильяма Остина (1754–1793) о составе аммиака (80% азота и 20% водорода) Дальтон определил относительную атомную массу азота, равную 4 (также в предположении о равном числе атомов водорода и азота в этом соединении).

А из данных по анализу некоторых углеводородов Дальтон приписал углероду значение 4,4. В 1803 Дальтон составил первую в мире таблицу относительных атомных масс некоторых элементов. В дальнейшем эта таблица претерпела очень сильные изменения; основные из них произошли еще при жизни Дальтона, что видно из следующей таблицы, в которой приведены данные из учебников, изданных в разные годы, а также в официальном издании ИЮПАК – Международного союза теоретической и прикладной химии (International Union of Pure and Applied Chemistry).

Таблица 1.
Элемент ННеСNО
Дальтон, 180314,545,66
Бецелиус, 1826112,2614,1816,02
Жерар, 18421121416
Менделеев, 19061,0084,012,014,0416,000
Бьеррум, 19331,0074,00212,014,00816,0000
Сиборг, 19451,0084,00312,01014,00816,000
ИЮПАК, 19931,007944,00260212,01114,0067415,9994

Прежде всего, обращают на себя внимание непривычные атомные массы у Дальтона: они в несколько раз отличаются от современных! Это объясняется двумя причинами. Первая – неточность эксперимента в конце 18 – начале 19 в. Когда Гей-Люссак и Гумбольдт уточнили состав воды (12,6% Н и 87,4% О), Дальтон изменил значение атомной массы кислорода, приняв ее равной 7 (по современным данным в воде 11,1% водорода). По мере совершенствования методов измерения уточнялись атомные массы и многих других элементов. При этом за единицу измерения атомных масс сначала выбирали водород, потом – кислород, а в настоящее время – углерод.

Вторая причина более серьезная. Дальтон не знал, в каком соотношении находятся атомы разных элементов в различных соединениях, поэтому он принял наиболее простую гипотезу о соотношении 1:1. Так считали многие химики, пока не были надежно установлены и приняты химиками правильные формулы для состава воды (Н2О) и аммиака (NH3), многих других соединений. Для установления формул газообразных веществ использовался закон Авогадро, позволяющий определять относительную молекулярную массу веществ.

Для жидких и твердых веществ использовали другие способы (см. МОЛЕКУЛЯРНОЙ МАССЫ ОПРЕДЕЛЕНИЕ). Особенно просто было устанавливать формулы соединений элементов переменной валентности, например, хлорида железа. Относительная атомная масса хлора уже была известна из анализа ряда его газообразных соединений. Теперь, если принять, что в хлориде железа число атомов металла и хлора одинаково, то для одного хлорида относительная атомная масса железа получалась равной 27,92, а для другого – 18,62. Отсюда следовало, что формулы хлоридов FeCl
2
и FeCl3, и Ar(Fe) = 55,85 (среднее из двух анализов). Вторая возможность – формулы FeCl4 и FeCl6, и Ar(Fe) = 111,7 – была исключена как маловероятная. Относительные атомные массы твердых веществ помогало находить эмпирическое правило, сформулированное в 1819 французскими учеными П.И.Дюлонгом и А.Т.Пти: произведение атомной массы на теплоемкость – величина постоянная. Особенно хорошо правило Дюлонга – Пти выполнялось для металлов, что позволило, например, Берцелиусу уточнить и исправить атомные массы некоторых из них.

При рассмотрении относительных атомных масс химических элементов, приводящихся в периодической таблице, можно заметить, что для разных элементов они даются с разной точностью. Например, для лития – с 4 значащими цифрами, для серы и углерода – с 5, для водорода – с 6, для гелия и азота – с 7, для фтора – с 8. Отчего такая несправедливость?

Оказывается, точность, с которой определяется относительная атомная масса данного элемента, зависит не столько от точности измерений, сколько от «природных» факторов, не зависящих от человека. Они связаны с непостоянством изотопного состава данного элемента: в разных образцах соотношение изотопов не вполне одинаковое. Например, при испарении воды молекулы с легкими изотопами (

см. ЭЛЕМЕНТЫ ХИМИЧЕСКИЕ) водорода переходят в газовую фазу чуть быстрее, чем молекулы тяжелой воды, содержащие изотопы 2Н. В результате в водяных парах изотопа 2Н немного меньше, чем в жидкой воде. Многие организмы также разделяют изотопы легких элементов (для них разница в массах более существенна, чем для тяжелых элементов). Так, при фотосинтезе растения отдают предпочтение легкому изотопу 12С. Поэтому в живых организмах, а также произошедших от них нефти и угле содержание тяжелого изотопа 13С понижено, а в углекислом газе и образовавшемся из него карбонатах, наоборот, – повышено. Микроорганизмы, восстанавливающие сульфаты, также накапливают легкий изотоп 32S, поэтому в осадочных сульфатах его больше. В «остатках» же, не усвоенных бактериями, доля тяжелого изотопа 34S больше. (Кстати, анализируя соотношение изотопов серы, геологи могут отличить осадочный источник серы от магматического. А по соотношению изотопов 12С и 13С можно даже отличить тростниковый сахар от свекловичного!)

Итак, для многих элементов приводить очень точные значения атомных масс просто не имеет смысла, поскольку они немного меняются от одного образца к другому. По точности, с какой приводятся атомные массы, можно сразу сказать, происходит ли в природе «разделение изотопов» данного элемента и насколько сильно. А вот, например, для фтора атомная масса приводится с очень высокой точностью; значит, атомная масса фтора в любом его земном источнике постоянна. И это неудивительно: фтор относится к так называемым элементам-одиночкам, которые в природе представлены одним-единственным нуклидом.

В периодической таблице массы некоторых элементов стоят в скобках. Это относится главным образом к актинидам, стоящим после урана (так называемые трансурановые элементы), к еще более тяжелым элементам 7-го периода, а также к нескольким более легким; среди них технеций, прометий, полоний, астат, радон, франций. Если сравнить таблицы элементов, напечатанные в разные годы, то окажется, что эти числа время от времени меняются, иногда в течение всего нескольких лет. Некоторые примеры приведены в таблице.

Причина изменений в таблицах заключается в том, что указанные элементы радиоактивны, у них нет ни одного стабильного изотопа. В таких случаях принято приводить либо относительную атомную массу наиболее долгоживущего нуклида (например, для радия), либо массовые числа; последние приводятся в скобках. Когда открывают новый радиоактивный элемент, то получают вначале лишь один из многих его изотопов – конкретный нуклид с определенным числом нейтронов. Исходя из теоретических представлений, а также экспериментальных возможностей, стараются получить нуклид нового элемента с достаточным временем жизни (с таким нуклидом легче работать), однако удавалось это «с первого захода» не всегда. Как правило, при дальнейших исследованиях выяснялось, что существуют и могут быть синтезированы новые нуклиды с бoльшим временем жизни, и тогда проставленное в Периодической таблице элементов Д.И.Менделеева число надо было заменять. Сопоставим массовые числа некоторых трансуранов, а также прометия, взятые из книг, изданных в разные годы. В скобках в таблице приведены современные данные для периодов полураспада. В старых изданиях вместо принятых в настоящее время символов элементов 104 и 105 (Rf – резерфордий и Db – дубний) фигурировали Ku – курчатовий и Ns – нильсборий.

Таблица 2.
Элемент ZГод издания
1951195819832000
Pm   61147 (2,62 года)145 (18 лет)145145
Pu    94239 (24100 лет)242 (3,76.105 лет)244 (8,2.107 лет)244
Am    95241 (432 года)243 (7370 лет)243243
Cm    96242 (163 сут)245 (8500 лет)247 (1,58.107 лет)247
Bk    97243 (4,5 час)249 (330 сут)247 (1400 лет)247
Cf    98245 (44 мин)251 (900 лет)251251
Es    99254 (276 сут)   254252 (472 сут)
Fm    100253 (3 сут)257 (100,5 сут)257
Md    101256 (76 мин)258 (52 сут)258
No    102255 (3,1 мин)259 (58 мин)
Lr    103256 (26 сек)262 (3,6 час)
Rf    104261 (78 сек)261
Db    105261 (1,8 сек)262 (34 сек)

Как видно из таблицы, все приведенные в ней элементы радиоактивные, их периоды полураспада намного меньше возраста Земли (несколько млрд. лет), поэтому в природе этих элементов нет и получены они искусственно. По мере совершенствования техники эксперимента (синтез новых изотопов и измерение времени их жизни) иногда удавалось найти нуклиды, живущие в тысячи и даже миллионы раз дольше известных до этого. Например, когда в 1944 на циклотроне в Беркли были поставлены первые опыты по синтезу элемента № 96 (впоследствии его назвали кюрием), то единственная имевшаяся тогда возможность получения этого элемента заключалась в облучении a-частицами ядер плутония-239: 239Pu + 4He ® 242Cm + 1n. Полученный нуклид нового элемента имел период полураспада около полугода; он оказался очень удобным компактным источником энергии, и позднее его использовали с этой целью, например, на американских космических станциях «Сервейор». В настоящее время получен кюрий-247, который имеет период полураспада 16 млн. лет, что в 36 млн. раз превышает время жизни первого известного нуклида этого элемента. Так что изменения, вносимые время от времени в таблицу элементов, могут быть связаны не только с открытием новых химических элементов!

В заключение – о том, как узнали, в каком соотношении присутствуют в элементе разные изотопы? Например, о том, что в природном хлоре на долю 35Cl приходится 75,77% (остальное – изотоп 37Cl)? В данном случае, когда в природном элементе всего два изотопа, решить задачу поможет такая аналогия.

В 1982 в результате инфляции стоимость меди, из которых чеканились одноцентовые монеты США, превысила номинал монеты. Поэтому с этого года монеты делают из более дешевого цинка и лишь сверху покрывают тонким слоем меди. При этом содержание дорогой меди в монете снизилось с 95 до 2,5%, а масса – с 3,1 до 2,5 г. Через несколько лет, когда в обращении находилась смесь монет двух типов, преподаватели химии сообразили, что эти монеты (на глаз они почти неразличимы) – прекрасное пособие для их «изотопного анализа», либо по массе, либо по числу монет каждого типа (аналогия массовой или мольной доли изотопов в смеси). Будем рассуждать так: пусть у нас имеется 210 монет, среди которых есть и легкие, и тяжелые (это соотношение не зависит от числа монет, если их достаточно много). Пусть также общая масса всех монет равна 540 г. Если бы все эти монеты были «легкой разновидности», то общая их масса была бы равна 525 г, что на 15 г меньше действительной. Почему так? Потому что не все монеты легкие: есть среди них и тяжелые. Замена одной легкой монеты на тяжелую приводит к увеличению общей массы на 0,6 г. Нам же надо увеличить массу на 40 г. Следовательно, легких монет имеется 15/0,6 = 25. Таким образом, в смеси 25/210 = 0,119 или 11,9% легких монет. (Конечно, со временем «изотопное соотношение» монет разного типа будет меняться: легких будет все больше, тяжелых – все меньше. Для элементов же соотношение изотопов в природе постоянно.)

Точно так же и в случае изотопов хлора или меди: известна средняя атомная масса меди – 63,546 (ее определили химики, анализируя различные соединения меди), а также массы легкого 64Cu и тяжелого 65Cu изотопов меди (эти массы определили физики, используя свои, физические, методы). Если элемент содержит более двух стабильных изотопов, их соотношение определяется другими методами.

Наши монетные дворы – Московский и Санкт-Петербургский тоже, оказывается, чеканили разные «изотопные разновидности» монет. Причина та же – подорожание металла. Так, 10- и 20-рублевые монеты в 1992 чеканились из немагнитного медно-никелевого сплава, а в 1993 – из более дешевой стали, и эти монеты притягиваются магнитом; по внешнему виду они практически не различаются (кстати, часть монет этих годов отчеканены «не в том» сплаве, такие монеты очень редкие, а некоторые стоят дороже золота!). В 1993 чеканились также 50-рублевые монеты из медного сплава, и в том же году (гиперинфляция!) – из стали, покрытой латунью. Правда, массы наших «изотопных разновидностей» монет отличаются не так сильно, как у американских. Тем не менее, точное взвешивание кучи монет дает возможность рассчитать, сколько в них монет каждого сорта – по массе, либо по числу монет, если подсчитано общее их число.

Илья Леенсон

КАК ОПРЕДЕЛИТЬ АТОМНУЮ МАССУ АЗОТА видео онлайн

Видео:

Относительная атомная масса – это масса атома, выраженная в атомных единицах массы (а. е. м.).В этом уроке

ВНИМАНИЕ! При оплате первого урока второй в подарок! Сегодня поговорим об относительной атомной и молек

В этом уроке ты узнаешь, что такое абсолютная и относительная масса атома, как рассчитывается относител

номер карты для поддержки канала 4102321254872814 или мой кошелек QIWI +380509793621 В данном уроке ты узнаешь . ..

Понятие относительной плотности газов с примерами его использования в расчетах. Задачи по органическо

УрокиПоХимии #ChemEra #ЗадачиПоХимии #ХимическиеУравнения #МассаХимия #ОбъемХимия #КоличествоВещества …

Изучение понятий «валентность» и «степень окисления» ― залог успешных знаний в химии. Урок позволяет .

Таблица Менделеева и таблица растворимости ru/himiya/56932-demoversiya-ege-2019-po-himii.html.

Понятие относительности вводится в химии для того, чтобы сделать расчеты проще. это и в самом деле необх

Ты, должно быть, понимаешь, что такое доля (она же часть). Выразить долю можно по-разному: в виде обыкновен

На сайте школьники могут подготовиться к ЕГЭ, ОГЭ и олимпиадам и улучшить знания по школьным предметам.

молярнаямасса #MEKTEП_OnLine #MEKTEP_OnLine МЕКТЕП OnLine — образовательный проект! Татьяна Белоусова …

Готовимся к практическому туру Всероссийской олимпиады по биологии!

Полную текстовую версию урока можно посмотреть на нашем сайте по ссылке: …

Относительная молекулярная масса Относительная атомная масса …

В этом уроке Вы узнаете про относительную молекулярную и относительную формульную массу, как они рассч

Массовая доля элемента в сложном веществе На этом уроке ты узнаешь, что такое массовая доля элемента в с

Молярная масса оксида азота 1

Анаболические стероиды — препараты, синтезированные на базе мужского полового гормона тестостерона. Одни из них (например, неробол) применяют в виде таблеток, другие (например, ретаболил, нероболил) являются средствами пролонгированного то есть продлённого действия и предназначены для внутримышечных инъекций. Основное свойство этих препаратов — усиление процесса обмена и усвоения тех веществ, которые идут на построение тканей живого организма, с одновременным ослаблением реакций обмена, связанных с распадом сложных органических веществ. Благодаря этому свойству их называют «строительными» гормонами. Причем в первую очередь препараты стимулируют белковый обмен. Анаболические стероиды активизируют также минеральный обмен, задерживая в организме калий, фосфор и серу, необходимые для синтеза белка, способствуют задержке кальция в костях.

Питание перед тренировкой

Питание перед тренировкой должно содержать углеводы, белки и совсем не содержать жиров (желательно, не больше 3 грамм).

Углеводы перед тренировкой необходимы для того, чтобы загрузить гликогеновые закрома и обеспечить мышцы и мозг энергией. Во время тренировок топливо сжигается очень быстро, и нужно, чтоб оно было гликогеновым, так как из жира тело не может поставлять нужные количества энергии (из-за нехватки кислорода).

Белки перед тренировкой не будут источником энергии, они будут источником аминокислот для работающих мышц. В результате сразу после тренировки синтез белка в мышцах резко возрастает.

Жир в питании перед тренировкой должен отсутствовать, потому что жир в пище замедляет опорожнение желудка и скорость пищеварения. Жирная пища дольше находится в желудке, и если она там будет находится во время тренировки, то может вызвать колики, тошноту и отрыжку.

Классическими приемами пищи перед тренировкой будут следующие:

• мясо птицы (индюшка, куриные грудки) с грубым хлебом или рисом

• нежирный бифштекс с картофелем

• омлет из белков яиц с овсянкой

Калорийность приема пищи должна быть обычной, такой же, как и у всех других ваших приемов пищи. Объемную пищу (большую порцию салата или тарелку супа) лучше съесть за час-два до тренировки, чтобы она успела перевариться, и желудок опустел. Более плотную пищу (полтарелки каши или творожка) можно съесть за 30 минут-час до начала тренировки.

Если вы тренируетесь, чтобы нарастить мышечную массу, то за 30 минут до тренировки съешьте один фрукт крупных размеров с низким гликемическим индексом (яблоко, груша, клубника или любые другие ягоды) и запейте его белковым напитком (лучше из сывороточного белка (whey protein powder)). Расчет белка в этом коктейле следующий: 0.22 г сывороточного белка на килограмм веса. Например, если вы весите 68 кг, то в коктейле (замешанном на воде) должно быть 15 г белка.

Также за 30 минут до тренировки выпейте стакан крепкого черного кофе (можно с сахарозаменителем, но не со сливками) или очень крепкого зеленого чая. Это поможет секреции эпинефрина и норэпинефрина, которые мобилизуют жир из жировых клеток, чтобы тело могло воспользоваться им как топливом. Таким образом, во время тренировки вы сожжете больше жира и меньше глюкозы, гликогена и аминокислот. Усталость в процессе тренировки наступит гораздо позже. Голова будет лучше соображать, и вы сможете тренироваться более интенсивно. Эффект от кофе перед тренировкой длится примерно 2 часа.

Сразу перед тренировкой лучше все-таки ничего не есть, так как физическая активность отвлекает от процесса пищеварения (ритмичных сокращений желудка, чтобы переварить пищу). В крайнем случае, если вы очень голодны, можно выпить стакан белкового коктейля или молока.

Питание во время тренировки

Самое главное во время тренировки — это не забывать пить! Уже при 2%-ном обезвоживании тренировка будет вялой и малоэффективной.

Не ориентируйтесь на чувство жажды. Интенсивные тренировки подавляют работу рецепторов жажды в горле и ЖКТ, так что к моменту, когда вам захочется пить, ваше тело уже будет обезвожено сверх меры. Кроме того, с возрастом датчики жажды в теле утрачивают свою чувствительность. Взрослым людям надо пить воду, потому что надо, а не потому, что хочется.

Если вы заметили симптомы обезвоживания (два или больше одновременно):

• чувство жажды

• сухость во рту

• сухие или даже потрескавшиеся губы

• головокружение

• усталость

• головная боль

• раздражительность

• отсутствие аппетита,

немедленно начинайте пить воду и прервите тренировку на несколько минут, пока не пройдут симптомы.

Режим питья следующий: прямо перед началом тренировки выпейте стакан воды и во время занятий пейте по чуть-чуть каждые 15-20 минут. Объем выпитого будет зависеть от количества пота. Вам нужно обеспечить гидрацию и даже супергидрацию организма во время тренировок.

Если тренировка длится больше часа, то желательтельно пить специальные спортивные напитки. С сахарами из них должно поступать примерно 30-60г углеводов в час. Больше 60г углеводов тело во время тренировки не усвоит, а продуктивность тренировки может снизиться. Пить калорийные напитки следует понемногу, отпивая каждые 10 минут. Спортивные напитки также содержат полезные электролиты (соли), которые тело утрачивает с потом и мочой.

Во время тренировки также можно пить фруктовые соки, желательно свежевыжатые, а не магазинные. Можно с уверенностью сказать, что все покупные соки, даже те, которые продаются с пометкой «100% сок без добавления сахара», разведены водой и содержат подмешанные сахара. Апельсиновые соки чаще всего содержат свекольный сахар, яблочные — кукурузный сироп и инулин. Самым лучшим соком является свежевыжатый апельсиновый, разведенные водой в пропорции 1:1.

Питание после тренировки

Есть надо сразу после тренировки, желательно, в первые 20 минут. Если воздерживаться от пищи в течение 2 часов после окончания тренировки, то тренировка теряет всякий смысл — в результате НИЧЕГО НЕ ТРЕНИРУЕТСЯ, немного сожжется жир и все, но прироста в силе, плотности мышц, стройности и скорости обмена веществ не будет. В первые 20 минут после тренировки в организме открыто так называемое послетренировочное (анаболическое) окно для потребления белков и углеводов (но не жиров). Все, что будет съедено в этот период, пойдет на восстановление мышц и прирост мышечной массы, ни одной калории из пищи не пойдет на жир. Это очень важно.

Углеводы после тренировки лучше потреблять в жидком виде из простых, выскокогликемических источников. Вам нужно добиться резкого скачка в уровне инсулина, с его анаболическими и антикатаболическими свойствами. Самыми лучшими считаются клюквенный и виноградный сок, потому что в них высокое соотношение глюкозы к фруктозе в углеводном профиле. Потребляйте примерно 1 г углеводов из сока на каждый килограмм ИДЕАЛЬНОГО веса. Стакан виноградного сока содержит 38 г углеводов (155 ккал), а стакан клюквенного — 31г углеводов (115 ккал). Также можно есть любую углеводную пищу, не содержашую жира (хлеб, варенье, сахар, картофель, рис, макароны, фрукты, овощи и т.д.).

Кроме того, сразу после тренировки нужно загрузиться белками. Лучше всего в форме белкового напитка из порошка. Таким способом, синтез белка в мышцах после тренировки увеличится в три раза (по сравнению с голоданием). Так что берите с собой бутылку с коктейлем из белкового порошка и сока, если вы тренируетесь вне дома, и выпейте все сразу, как только прекратите тренировку. Количество белка из порошка должно быть 0.55г на каждый килограмм идеального веса. Если вы не можете пить белковые коктейли по каким-то причинам, полагайтесь на белки яиц. Если есть возможность поесть в течение часа после тренировки, то выбирайте любую белковую пищу, просто рассчитайте нужное количество белка.

Поскольку у питания после тренировки есть только одна важная цель — максимально быстро и эффективно поспособствовать приросту мышечной массы, — то жира в этом приеме пищи не должно содержаться вообще. Жир в пище замедлит проход углеводов и белков из желудка в кровь. Белковая пища должна быть нежирной, т.е. если курица — то грудки, а не ножки. Если яйца, то только белки. Говядины и свинины следует избегать, так как они всегда очень жирные, отдавайте предпочтение телятине. Также надо быть осторожными с сыром, молоком, йогуртами и творогом — как правило, они содержат в себе не меньше 5% жира. Исключением является только жирная рыба (не жареная!). Ее можно и нужно есть как можно чаще.

После тренировки, в течение двух часов, желательно исключить все, что содержит кофеин: кофе, чай, какао и все «шоколадное» (даже белковые порошки со вкусом шоколада). Дело в том, что кофеин вмешивается в работу инсулина и, таким образом, мешает вашему телу перезагрузить гликоген в мышцы и печень и воспользоваться белком для ремонта мышц. Так что если вы тренируетесь по утрам, терпите 2 часа, а уж потом пейте настоящий крепкий кофе. Чашка кофе, выпитая перед тренировкой, должна помочь вам оставаться бодрыми и энергичными. Если совсем не можете отказаться от кофе или чая, выбирайте их декафинизированные аналоги.

Не тренируйтесь зря — выжимайте максимум эффекта из минимума затрат путем оптимизации питания до, во время и после тренировки!

Молярная масса оксида азота 1. Testocypol

Применять стевию как натуральный сахарозаменитель рекомендовано всем взрослым, пожилым, детям, здоровым людям и тем, которые болеют. Заказать с доставкой Туринабол Индийский Давлеканово Как получить скидку на Сустанон Balkan Pharmaceuticals Вольск Как узнать цену Tимозин Альфа St Biotechnology Радужный Авторские статьи получать оксид азота 3 формула новые статьи автора на почту Сортировать по Дате публикации Рейтингу Популярности Все разделы Тренировки Питание Добавки Мотивация Комментарии 10. Заказать с доставкой Станаболик Balkan Pharmaceuticals Смоленск Как заказать Суспензия тестостерона Balkan Pharmaceuticals Елизово Не так давно мы видели ее пышные формы, но уже сегодня она стройная и роскошная. Какая дозировка у Дека Дураболин Lyka labs Арамиль Для лиц старше 16 лет Александр Муромский. Заказать дешево молярной массы оксида азота 1 Дураболин Balkan Pharmaceuticals Свободный Правда, пока такие таблетки попадались растворимые, без твердых оболочек.

Применительно к телесным и духовным практикам оно прозвучала у поволжской целительницы Арины. При все при этом делать макароны по-флотски так же не хочется. Заказать дешево Болденол SP Laboratories Кизел В процессе укладки предпочтительнее пользоваться не просто феном, а феном с диффузором.

Где купить со скидкой Тренболон British Dispensary Шахунья Купить дешевле Кленбутерол Body Pharm Навашино Здесь можно купить Гексарелин St Biotechnology Юрьевец Как получить скидку на Тритрен SP Laboratories Слюдянка Где купить дешево Мастерон Lyka Labs Анадырь Заказать со скидкой Тренболон Golden Dragon Междуреченск Здесь можно купить Тестостерон Ципионат SP Laboratories Наро-Фоминск Где купить со скидкой Азолол British Dispensary Камышлов Где узнать цену CJC 1295 St Biotechnology Болохово Где получить скидку на Oxandrol Lyka Labs Нефтекумск Во время формирования пола у плода млекопитающих тестостерон приводит к маскулинизации структур Вольфа и вызывает формирование внешних гениталий в виде мошонки и пениса. На вид бледноват,но по описанию и подбору продуктов должен получится вкусным.

Как принимать Дека Дураболин Lyka labs Исилькуль Как использовать Trenbolone Body Pharm Слюдянка Тел 89277017171 дорогие девочки! Где купить Метандиенон RADJAY Лесогорск Как заказать и принимать молярная масса оксида азота 1 Дураболин Vermodje Суоярви Здесь можно купить Винстрол Brirish Dispensary Байкальск Здесь крем aqua peptide купить Андролик British Dispensary Мариинск Здесь можно купить Гонадотропин Ferring GMBH Гусев Розовый монумент высотой 8,5 метра будет установлен на берегу Балтийского моря. Как употреблять Оксандролон British Dragon Чебаркуль Как употреблять Дека Дураболин Lyka labs Мосальск В то время я еще мониторил вашу тему (пока вы не скатились в гантельно-гиревую зарядку). Даже хорошо, что турнир в Финляндии станет оксид азота в организме для него третьим.

Купить дешевле IGF-1 St Biotechnology Вихоревка Яичный шампунь для сухих, окрашенных волос. Здесь можно купить Тритренол Lyka Labs Судогда Купить дешевле Тестостерон Пропионат SP Laboratories Усть-Лабинск Где узнать цену Дека Дураболин Balkan Pharmaceuticals Козловка Где продается Болденона Ундесиленат Lyka Labs Лесосибирск Вкусного тортика и большое спасибо за внимание!!!

Где узнать цену Тренболон Lyka Labs Чудово Как принимать Анастровер Vermoje Алдан Напомню Ты говорил, что при отжимании на брусьях, надо корпус наклонять вперед, если прокачиваешь грудь. Как заказать Сустанон Organon Голландский Алексеевка Как мне взять Тестостерон Энантат Vermoje Ядрин Где получить скидку на Параболан Balkan Pharmaceuticals Красноборск И конечно, необходима вера в собственные силы. Заказать с доставкой Станозолол Balkan Pharmaceuticals Электроугли Где узнать цену PEG MGF St Biotechnology Каменск-Уральский Как я могу купить Кленбутерол SP labs Суровикино Где купить дешево Sustanon Body Pharm Ужур В этот раз на тренировку я принёс еще одну Супер Катану, тоже 54-го размера, черного цвета, практически новую и обычную Катану, но уже 52-го размера. Заказать по низкой цене Сустанон Пакистанский Кириллов Где купить дешево Сустанон Пакистанский Арсеньев Как заказать Тестостерон Ципионат Lyka labs Воткинск получение оксида азота 3 Где купить со скидкой Тестостерон Энантат Lyka Labs Протвино Где купить со скидкой Хорагон Ferring GMBH Серов Как я могу купить Дека Дураболин Balkan Pharmaceuticals Набережные Челны Какой эффект от приема Туринабол British Dragon Клин Где заказать Сайзен EMD Serono Ковдор Заказать со скидкой Примобол Balkan Pharmaceuticals Скопин Я здесь замучилась бороться с молярными массами оксида азота 1… Как заказать Метандиенон SP Labolatories Рыбное Где купить Тритрен SP Laboratories Магнитогорск Да я, как бы, и умела вставлять, просто писать сообщение, а потом заходить на сайт, выбирать анимашки и снова в сообщение вернуться, чтобы вставить… Даже если вы следуете последним тенденциям моды, не всегда получается нужный результат. И где тепрь найти, не знаю Зайчиришка Ириша 33 года Москва 15 Дек 2014 1820 Зайчиришка писала магазина нет теперь В интернете заказывайте, тем более, уже знаете проверенные средства. Купить дешевле Дростанолон Golden Dragon Зеленокумск Заказать дешево Данабол Balkan Pharmaceuticals Вытегра Как узнать цену Станозолол British Dispensary Сибай Все подобные рецепты всегда откладываются до лучших времен,когда нас будет много Жанчик oracle stack trace И оно всегда делается с миндальной мукой или мукой других орехов А есть еще пропитываются сиропом..

И упустила момент,что нужно партиями жарить,обжаривала сразу все баклажаны. Купить дешевле Винстрол Vermoje Ворсма Заказать дешево Кломид Balkan Pharmaceuticals Верхний Уфалей Заказать с доставкой Гексарелин St Biotechnology Горячий Ключ Как мне взять Тестостерон Энантат ABURAIHAN IRAN Козловка Как принимать Нандролона деканоат Голландский Сосновый Бор Как заказать GHRP-2 St Biotechnology Чудово Как мне купить Тренболон Ацетат Body Pharm Константиновск Здесь можно купить Диноджет Golden Dragon Дагестанские Огни Как принимать Тестостерон Пропионат Body Pharm Сходня Купить дешевле Тамоксифен Balkan Pharmaceuticals Туринск Их используют в том числе и в клинике, эти препараты обладают и масса космического аппарата равна 2000 лечебными свойствами и магазин спортивного питания самара определенные показания, когда людям эти препараты назначают.


Аммиак — Что такое Аммиак?

Аммиак представляет собой бинарное неорганическое химическое соединение азота и водорода

Аммиак (нитрид водорода) — химическое соединение азота и водорода с формулой Nh4, при нормальных условиях — бесцветный газ с резким характерным запахом.

Плотность аммиака почти в 2 раза меньше, чем у воздуха, ПДКр.з. 20 мг/м3 — IV класс опасности (малоопасные вещества) по ГОСТ 12.1.007.
Растворимость Nh4 в воде чрезвычайно велика — около 1200 объемов (при 0°C) или 700 объемов (при 20°C) в объеме воды.
В холодильной технике носит название R717, где R — Refrigerant (хладагент), 7 — тип хладагента (неорганическое соединение), 17 — молекулярная масса.

Аммиак относится к числу важнейших продуктов химической промышленности, ежегодное его мировое производство превышает 180 млн т.

Молекула аммиака имеет форму треугольной пирамиды с атомом азота в вершине. 3 неспаренных p-электрона атома азота участвуют в образовании полярных ковалентных связей с 1s-электронами 3х атомов водорода (связи N − H), 4я пара внешних электронов является неподеленной, она может образовать ковалентную связь по донорно-акцепторному механизму с ионом водорода, образуя ион аммония Nh5+.
Несвязывающее 2х-электронное облако строго ориентировано в пространстве, поэтому молекула аммиака обладает высокой полярностью, что приводит к его хорошей растворимости в воде.

В жидком аммиаке молекулы связаны между собой водородными связями. Сравнение физических свойств жидкого аммиака с водой показывает, что аммиак имеет более низкие температуры кипения (tкип −33,35 °C) и плавления (tпл −77,70 °C), а также меньшие плотность, вязкость (в 7 раз меньше вязкости воды), проводимость (почти не проводит электрический ток) и диэлектрическую проницаемость.
Это в некоторой степени объясняется тем, что прочность водородных связей в жидком аммиаке существенно ниже, чем у воды; а также тем, что в молекуле аммиака имеется лишь одна пара неподеленных электронов, в отличие от двух пар в молекуле воды, что не дает возможность образовывать разветвленную сеть водородных связей между несколькими молекулами.
Аммиак легко переходит в бесцветную жидкость с плотностью 681,4 кг/м3, сильно преломляющую свет.

Подобно воде, жидкий аммиак сильно ассоциирован, главным образом за счет образования водородных связей.
Жидкий аммиак — хороший растворитель для очень большого числа органических, а также для многих неорганических соединений.
Твердый аммиак — кубические кристаллы.

18.12 Вперше в історії атомні маси элементів будуть змінені

Статья с ресурса http://www.chemport.ru/datenews.php?news=2310

 

http://www.iupac.org/web/nt/2010-12-13_2009-atomic-weights

 

Впервые в истории атомные массы некоторых элементов, приведенные в Периодической системе Д.И. Менделеева будут изменены – такое изменение должно коснуться бесчисленного количества таблиц Менделеева, висящих во всем мире на стенах химических аудиторий и лабораторий, а также напечатанных в учебниках.

В «новой» таблице, данные для корректировки некоторых атомных весов которой были недавно опубликованы в материалах Международного Союза по теоретической и прикладной химии, (IUPAC) должны измениться атомные массы и форма их представления для десяти элементов – водорода, лития, бора, углерода, азота, кислорода, кремния, серы, хлора и таллия. Новые значения атомных масс более точно отражают изотопное распределение этих элементов в природе.

Как отмечает Майкл Визер (Michael Wieser) из Университета Калгари, секретарь комиссии IUPAC по атомным массам и распространенности изотопов, изучающей распространенность различных изотопов в земной коре и уточняющей значения атомных масс, почти полтора столетия мы пользовались стандартным набором значений атомных весов, определённых еще до таблицы Менделеева или скорректированных при ее создании. Однако аналитические технологии не стоят на месте, и во второй половине ХХ века стало ясно, что атомная масса не является статической для всей земной коры величиной, а зависит зачастую от места извлечения того или иного элемента.

Современные аналитические технологии могут осуществлять точные изменения атомных масс многих элементов, при этом незначительные изменения в значениях атомной массы очень часто могут оказаться важными как для решения исследовательских задач, так и на практике. Например, точное определение распространенности изотопов углерода в образце может использоваться для определения качества пищи или особенностей ее происхождения. Изотопное распределение азота, хлора и ряда других элементов важно для отслеживания перемещения загрязняющих веществ в гидросфере и грунтовых водах. При проведении допинг-контроля различия в изотопном составе могут также оказаться полезными – распределение изотопов в тестостероне, вырабатывающимся организмом человека и фармацевтическом тестостероне различно.

Скорректированные атомные массы 10 элементов теперь будут выражаться в виде интервалов, для которых будет обозначаться наибольшее и наименьшее значение, соответствующее наиболее точно измеренному разбросу атомных масс. Например, для серы в таблице сейчас приводится значение атомной массы 32.065 Да, а в соответствии с новым требованием IUPAC — будет приводится интервал от 32.059 до 32.076 Да, отражающий, что относительная атомная масса зависит от источника элемента. Как отмечает Визер, точное значение атомной массы позволит определить происхождение элемента и его соединения.

Прикріплений файлРозмір
Atomic weights of the elements 2009 (IUPAC Technical Report)1.75 MB

3: Относительные атомные массы и эмпирические формулы

Foundation

Мы начнем с принятия центральных постулатов атомно-молекулярной теории . Это: элементы состоят из одинаковых атомов; все атомы одного элемента имеют одинаковую характеристическую массу; количество и массы этих атомов не меняются при химическом превращении; соединения состоят из идентичных молекул, состоящих из атомов, объединенных в простых целочисленных соотношениях. Мы также предполагаем, что знаем наблюдаемые законы природы, на которых основана эта теория: Закон сохранения массы , Закон определенных пропорций и Закон множественных пропорций .

Голы

Мы пришли к выводу, что атомы объединяются в простых соотношениях, образуя молекулы. Однако мы не знаем, что это за отношения. Другими словами, мы еще не определили никаких молекулярных формул. В таблице 2.2 мы обнаружили, что массовые отношения для соединений оксида азота соответствуют множеству различных молекулярных формул. Взгляд назад на данные по оксиду азота показывает, что оксид B может быть \ (\ ce {NO} \), \ (\ ce {NO_2} \), \ (\ ce {N_2O} \) или любым другим простым соотношением .

Каждая из этих формул соответствует различным возможным относительным атомным весам азота и кислорода.Поскольку оксид B имеет отношение кислорода к азоту 1,14: 1, то относительные массы кислорода к азоту могут составлять 1,14: 1 или 2,28: 1 или 0,57: 1 или многие другие простые возможности. Если бы мы знали относительные массы атомов кислорода и азота, мы могли бы определить молекулярную формулу оксида B. С другой стороны, если бы мы знали молекулярную формулу оксида B, мы могли бы определить относительные массы атомов кислорода и азота. Если мы решаем одну проблему, мы решаем обе. Наша проблема в том, что нам нужен простой способ «подсчитать» атомы, по крайней мере, в относительных числах.

Наблюдение 1: Объемные отношения в химических реакциях

Хотя масса сохраняется, большинство химических и физических свойств не сохраняется во время реакции. Объем — одно из тех свойств, которое не сохраняется, особенно когда в реакции используются газы в качестве реагентов или продуктов. Например, водород и кислород вступают во взрывную реакцию с образованием водяного пара. Если мы возьмем 1 литр газообразного кислорода и 2 литра газообразного водорода, путем тщательного анализа мы сможем обнаружить, что реакция этих двух объемов завершена, без остатка водорода и кислорода, и что образуется два литра водяного пара.Обратите внимание, что общий объем не сохраняется: 3 литра кислорода и водорода превращаются в 2 литра водяного пара. (Все объемы измеряются при одинаковой температуре и давлении.)

Более примечателен тот факт, что соотношения задействованных объемов представляют собой простые целочисленные соотношения: 1 литр кислорода: 2 литра водорода: 2 литра воды. Этот результат оказывается общим для реакций с участием газов. Например, 1 литр газообразного азота реагирует с 3 литрами газообразного водорода с образованием 2 литров газообразного аммиака.1 литр газообразного водорода объединяется с 1 литром газообразного хлора с образованием 2 литров газообразного хлористого водорода. Эти наблюдения могут быть обобщены в Законе объединения объемов .

Закон объединения объемов

Когда газы соединяются во время химической реакции при фиксированных давлении и температуре, отношения их объемов представляют собой простые целочисленные отношения.

Эти простые целочисленные отношения поразительны, особенно если рассматривать их в свете наших выводов из Закона множественных пропорций.Атомы соединяются в простых целочисленных отношениях, и, очевидно, объемы газов также объединяются в простых целочисленных отношениях. Почему это могло быть? Одно простое объяснение этого сходства состоит в том, что объемное соотношение и соотношение атомов и молекул в реакции одинаковы. В случае водорода и кислорода это означает, что соотношение объемов (1 литр кислорода: 2 литра водорода: 2 литра воды) такое же, как соотношение атомов и молекул (1 атом кислорода: 2 литра воды). атомы водорода: 2 молекулы воды).Чтобы это было правдой, равные объемы газа должны содержать равное количество частиц газа (атомов или молекул), независимо от типа газа. Если это так, это означает, что объем газа должен быть прямой мерой количества частиц (атомов или молекул) в газе. Это позволило бы «подсчитать» количество частиц газа и определить молекулярные формулы.

Однако этот вывод, похоже, вызывает большие проблемы. Посмотрите на данные по образованию хлористого водорода: из 1 литра водорода плюс 1 литр хлора получается 2 литра хлористого водорода.Если наше мышление верно, то это эквивалентно утверждению, что 1 атом водорода плюс 1 атом хлора составляют 2 молекулы хлористого водорода. Но как такое могло быть? Как мы могли сделать 2 идентичные молекулы из одного атома хлора и одного атома водорода? Это потребовало бы от нас разделить каждый атом водорода и хлора, нарушив постулаты атомно-молекулярной теории.

Другая проблема возникает, когда мы взвешиваем газы: 1 литр газообразного кислорода весит больше, чем 1 литр водяного пара.Если мы предположим, что эти объемы содержат равное количество частиц, то мы должны сделать вывод, что 1 частица кислорода весит больше, чем 1 частица воды. Но как такое могло быть? Казалось бы, молекула воды, содержащая хотя бы один атом кислорода, должна весить больше одной частицы кислорода.

Это серьезные возражения против идеи, что равные объемы газа содержат равное количество частиц. Наш постулат, похоже, противоречит здравому смыслу и экспериментальным наблюдениям.Однако не менее убедительны и простые соотношения Закона объединения объемов. Почему объемы должны реагировать в простых целочисленных отношениях, если они не представляют равное количество частиц? Рассмотрим противоположную точку зрения: если равные объемы газа не содержат равного количества частиц, то равное количество частиц должно содержаться в неравных объемах, не связанных целыми числами. Теперь, когда мы объединяем частицы в простых целочисленных соотношениях для образования молекул, требуемые объемы газов будут давать явно нецелочисленные отношения.Закону объединения объемов следует слегка противоречить. Есть только один логический выход. Мы примем вывод из Закона объединения объемов о том, что равных объемов газа содержат равное количество частиц , вывод, известный как Гипотеза Авогадро . Как учесть тот факт, что из 1 литра водорода плюс 1 литр хлора получается 2 литра хлористого водорода? Есть только один способ для одной частицы водорода произвести 2 идентичных молекулы хлористого водорода: каждая частица водорода должна содержать более одного атома.Фактически каждая частица (или молекула) водорода должна содержать четное число атомов водорода. Точно так же молекула хлора должна содержать четное число атомов хлора.

Более подробно, мы замечаем, что

\ [1 \: \ text {литр водорода} + 1 \: \ text {литр хлора} \ rightarrow 2 \: \ text {литры хлористого водорода} \]

Предполагая, что каждый литровый объем содержит равное количество частиц, мы можем интерпретировать это наблюдение как

\ [1 \ ce {H_2} \: \ text {молекула} + 1 \ ce {Cl_2} \: \ text {молекула} \ rightarrow 2 \ ce {HCl} \: \ text {молекулы} \]

(В качестве альтернативы, в каждой молекуле водорода и в каждой молекуле хлора может быть любое фиксированное четное число атомов.Мы предположим простейшую возможность и посмотрим, не приведет ли это к противоречиям.)

Это замечательный результат, поскольку он правильно учитывает Закон объединения объемов и устраняет наши опасения по поводу создания новых атомов. Самое главное, теперь мы знаем молекулярную формулу хлористого водорода. По сути, мы нашли способ «подсчета» атомов в реакции путем измерения объема реагирующих газов.

Этот метод позволяет нам узнать молекулярную формулу многих соединений.Например,

\ [2 \: \ text {литры водорода} + 1 \: \ text {литры кислорода} \ rightarrow 2 \: \ text {литры воды} \]

Для этого требуется, чтобы частицы кислорода содержали четное число атомов кислорода. Теперь мы можем интерпретировать это уравнение как говорящее, что

\ [2 \ ce {H_2} \: \ text {молекулы} + 1 \ ce {O_2} \: \ text {молекула} \ rightarrow 2 \ ce {H_2O} \: \ text {молекулы} \]

Теперь, когда мы знаем молекулярную формулу воды, мы можем сделать определенный вывод об относительных массах атомов водорода и кислорода.Напомним из таблицы 2.1, что массовое отношение кислорода к водороду в воде составляет 8: 1. Поскольку на каждый атом кислорода в воде приходится два атома водорода, то соотношение масс требует, чтобы один атом кислорода весил в 16 раз больше массы атома водорода.

Чтобы определить масштаб масс атомов, нам просто нужно выбрать стандарт. Например, для наших целей мы скажем, что атом водорода имеет массу 1 в атомной шкале масс. Тогда атом кислорода имеет массу 16 по этой шкале.

Наши выводы учитывают очевидные проблемы с массами реагирующих газов, в частности, то, что газообразный кислород весит больше, чем водяной пар.Это казалось бессмысленным: учитывая, что вода содержит кислород, казалось бы, что вода должна весить больше, чем кислород. Однако теперь это понятно: молекула воды, содержащая только один атом кислорода, имеет массу 18, тогда как молекула кислорода, содержащая два атома кислорода, имеет массу 32.

Определение атомного веса газообразных элементов

Теперь, когда мы можем подсчитывать атомы и молекулы для определения молекулярных формул, нам нужно определить относительные атомные веса для всех атомов.Затем мы можем использовать их для определения молекулярных формул любого соединения из массовых соотношений элементов в соединении.

Начнем с изучения данных о реакциях, связанных с законом объединения объемов. Возвращаясь к данным по оксиду азота, приведенным в Модуле 2, мы напоминаем, что есть три соединения, образованных из азота и кислорода. Теперь измеряем объемы, которые складываются в каждый. Мы обнаружили, что 2 литра оксида B можно разложить на 1 литр азота и 1 литр кислорода.Из приведенных выше рассуждений, частица азота должна содержать четное число атомов азота. Пока мы предполагаем, что азот — это \ (\ ce {N_2} \). Мы уже сделали вывод, что кислород — это \ (\ ce {O_2} \). Следовательно, молекулярная формула оксида B — \ (\ ce {NO} \), и мы называем его оксидом азота. Поскольку мы уже определили, что массовое отношение кислорода к азоту составляет 1,14: 1, то, если мы присвоим кислороду массу 16, как указано выше, масса азота будет равна 14. (То есть \ (\ frac {16} {1.14) } = 14 \).) 2 литра оксида А образуются из 2 литров кислорода и 1 литра азота.Следовательно, оксид A — это \ (\ ce {NO_2} \), который мы называем диоксидом азота. Обратите внимание, что мы прогнозируем массовое отношение кислорода к азоту \ (\ frac {32} {14} = 2,28: 1 \) в соответствии с данными. Оксид C представляет собой \ (\ ce {N_2O} \), называемый закисью азота, и, по прогнозам, имеет массовое отношение \ (\ frac {16} {28} = 0,57: 1 \), что опять же согласуется с данными. Теперь мы разрешили неоднозначность молекулярных формул.

Что, если бы азот на самом деле был \ (\ ce {N_4} \)? Тогда первый оксид будет \ (\ ce {N_2O} \), второй будет \ (\ ce {N_2O_2} \), а третий будет \ (\ ce {N_4O} \).Кроме того, масса атома азота будет 7. Почему мы этого не предполагаем? Просто потому, что при этом мы всегда обнаружим, что минимальная относительная масса азота в любой молекуле равна 14. Хотя это могут быть два атома азота, нет оснований полагать, что это так. Следовательно, один атом азота весит 14, а частицы газообразного азота равны \ (\ ce {N_2} \).

Определение атомного веса негазообразных элементов

Мы можем продолжить этот тип измерения, вывода и предсказания для любого соединения, которое является газом и которое состоит из элементов, являющихся газами.Но это не поможет нам с атомными массами негазообразных элементов и не позволит нам определить молекулярные формулы для соединений, которые содержат эти элементы.

Рассмотрим углерод, важный пример. Есть два оксида углерода. Оксид А имеет массовое отношение кислорода к углероду 1,33: 1, а оксид В имеет массовое отношение 2,66: 1. Измерение реакционных объемов показывает, что 1 литр оксида А образуется из 0,5 литра кислорода. Следовательно, каждая молекула оксида A содержит только половину атомов кислорода, чем молекула кислорода.Таким образом, оксид А содержит один атом кислорода. Но сколько в нем атомов углерода? Мы пока не можем определить это, потому что элементарный углерод твердый, а не газ. Это означает, что мы также не можем определить, какова масса атома углерода.

Но мы можем попробовать другой подход: мы взвешиваем 1 литр оксида А и 1 литр газообразного кислорода. В результате мы обнаружили, что оксид А весит 0,875 раза на литр больше, чем газообразный кислород. Поскольку мы предположили, что фиксированный объем газа содержит фиксированное количество частиц, то 1 литр оксида A содержит столько же частиц, сколько 1 литр газообразного кислорода.Следовательно, каждая частица оксида A весит в 0,875 раза больше, чем частица газообразного кислорода (то есть молекула \ (\ ce {O_2} \)). Поскольку молекула \ (\ ce {O_2} \) весит 32 по нашей атомной шкале масс, то частица оксида A весит \ (0,875 \ times 32 = 28 \). Теперь мы знаем молекулярную массу оксида А.

Кроме того, мы уже определили из объединенных объемов, что оксид A содержит единственный атом кислорода с массой 16. Следовательно, масса углерода в оксиде A равна 12. Однако на данный момент мы не знаем, является ли это атомом кислорода. атом углерода массы 12, два атома массы 6, восемь атомов массы 1.5 или одна из многих других возможностей.

Для дальнейшего прогресса мы проводим дополнительные измерения других углеродсодержащих газовых соединений. 1 литр оксида B углерода образуется из 1 литра кислорода. Следовательно, каждая молекула оксида B содержит два атома кислорода. 1 литр оксида B весит в 1,375 раза больше, чем 1 литр кислорода. Следовательно, одна молекула оксида B имеет массу \ (1,375 \ times 32 = 44 \). Поскольку в молекуле оксида B два атома кислорода, масса кислорода в оксиде B равна 32.Следовательно, масса углерода в оксиде B равна 12, как и в оксиде A.

Мы можем повторить этот процесс для многих таких газообразных соединений, содержащих атомы углерода. В каждом случае мы обнаруживаем, что масса углерода в каждой молекуле либо 12, либо кратна 12. Мы никогда не находим, например, 6 или 18), что было бы возможно, если бы каждый атом углерода имел массу 6. Простейший вывод состоит в том, что атом углерода имеет массу 12. Когда мы знаем атомную массу углерода, мы можем заключить, что молекулярная формула оксида A — \ (\ ce {CO} \), а оксида B — \ (\ ce { CO_2} \).

Следовательно, атомные массы негазообразных элементов могут быть определены путем измерения массы и объема газообразных соединений, содержащих эти элементы. Эта процедура является довольно общей, и таким образом можно определить большинство атомных масс.

Молекулы, молекулярные формулы и стехиометрические расчеты

Мы начали с круговой дилеммы: мы могли определять молекулярные формулы при условии, что мы знали атомные массы, но что мы могли определять атомные массы только на основе знания молекулярных формул.Поскольку теперь у нас есть метод определения всех атомных масс, мы решили эту дилемму и можем определить молекулярную формулу для любого соединения, для которого у нас есть процентный состав по массе.

В качестве простого примера рассмотрим соединение, которое состоит из \ (40,0 \% \) углерода, \ (53,3 \% \) кислорода и \ (6,7 \% \) водорода по массе. Вспомните из Закона определенных пропорций, что эти массовые отношения не зависят от образца, поэтому мы можем взять любой подходящий образец для проведения анализа.Предполагая, что у нас есть \ (100.0 \: \ text {g} \) соединения, мы должны иметь \ (40.0 \: \ text {g} \) углерода, \ (53.3 \: \ text {g} \) кислорода и \ (6.7 \: \ text {g} \) водорода. Если бы мы могли подсчитать или иным образом определить количество атомов каждого элемента, представленного этими массами, мы получили бы молекулярную формулу. Однако сделать это было бы не только чрезвычайно сложно, но и излишне.

Из нашего определения атомных масс мы можем отметить, что 1 атом углерода имеет массу, равную 12.0 масс атома водорода. Следовательно, масса \ (N \) атомов углерода также в 12,0 раз больше массы \ (N \) атомов водорода, независимо от того, что такое \ (N \). Если мы внимательно рассмотрим это, мы обнаружим, что \ (12.0 \: \ text {g} \) углерода содержит точно такое же количество атомов, что и \ (1.0 \: \ text {g} \) водорода. Точно так же отметим, что 1 атом кислорода имеет массу, которая в \ (\ frac {16.0} {12.0} \) раз больше массы атома углерода. Следовательно, масса \ (N \) атомов кислорода равна \ (\ frac {16.0} {12.0} \) умножить на массу \ (N \) атомов углерода. Опять же, мы можем сделать вывод, что \ (16.0 \: \ text {g} \) кислорода содержит точно такое же количество атомов, как \ (12.0 \: \ text {g} \) углерода, которое, в свою очередь, является тем же числом. атомов как \ (1.0 \: \ text {g} \) водорода. Не зная (или даже не заботясь об этом), что это за число, мы можем сказать, что оно одинаково для всех трех элементов.

Тогда для удобства мы определяем как количество атомов в \ (12.0 \: \ text {g} \) как 1 моль атомов.Обратите внимание, что 1 моль — это определенное количество частиц, так же как 1 дюжина — это определенное количество, независимо от того, какие объекты мы считаем. Преимущество такого определения моля заключается в том, что легко определить количество молей вещества, которое у нас есть, а знание количества молей эквивалентно подсчету количества атомов (или молекул) в образце. Например, \ (24.0 \: \ text {g} \) углерода содержит 2,0 моля атомов, \ (30.0 \: \ text {g} \) углерода содержит 2,5 моля атомов, и, как правило, \ ( x \) граммов углерода содержит \ (\ frac {x} {12.0} \) молей атомов. Также напомним, что \ (16.0 \: \ text {g} \) кислорода содержит ровно столько же атомов, сколько \ (12.0 \: \ text {g} \) углерода, и, следовательно, \ (16.0 \: \ text {g} \) кислорода содержит ровно 1,0 моль атомов кислорода. Таким образом, \ (32.0 \: \ text {g} \) кислорода содержит 2,0 моля атомов кислорода, \ (40.0 \: \ text {g} \) кислорода содержит 2,5 моля и \ (x \) граммов кислорода содержит \ (\ frac {x} {16.0} \) молей атомов кислорода. В более общем смысле, если у нас есть \ (m \) граммов элемента с атомной массой \ (M \), количество молей атомов \ (n \) равно

.

\ [n = \ frac {m} {M} \]

Теперь мы можем определить относительное количество атомов углерода, кислорода и водорода в нашем неизвестном выше соединении.В образце \ (100.0 \: \ text {g} \) у нас есть \ (40.0 \: \ text {g} \) углерода, \ (53.3 \: \ text {g} \) кислорода и \ (6.7 \: \ text {g} \) водорода. Таким образом, количество молей атомов в каждом элементе составляет

.

\ [\ begin {align} n_ \ ce {C} & = \ frac {40.0 \: \ text {g}} {12.0 \: \ frac {\ text {g}} {\ text {mol}}} \ \ & = 3.33 \: \ text {mol} \\ n_ \ ce {O} & = \ frac {53.3 \: \ text {g}} {16.0 \: \ frac {\ text {g}} {\ text { mol}}} \\ & = 3.33 \: \ text {mol} \\ n_ \ ce {H} & = \ frac {6.7 \: \ text {g}} {1.0 \: \ frac {\ text {g} } {\ text {mol}}} \\ & = 6.67 \: \ text {mol} \ end {align} \]

Отметим, что числа молей атомов элементов находятся в простом соотношении \ (n_ \ ce {C}: n_ \ ce {O}: n_ \ ce {H} = 1: 1: 2 \). Поскольку количество частиц в 1 моль одинаково для всех элементов, то также должно быть верно, что количество атомов элементов находится в простом соотношении 1: 1: 2. Следовательно, молекулярная формула соединения должна быть \ (\ ce {COH_2} \).

Или нет? При дальнейшем размышлении мы должны понять, что простое соотношение 1: 1: 2 не обязательно должно представлять точное количество атомов каждого типа в молекуле соединения, поскольку это действительно только соотношение.Таким образом, молекулярная формула может так же легко быть \ (\ ce {C_2O_2H_4} \) или \ (\ ce {C_3O_3H_6} \). Поскольку формула \ (\ ce {COH_2} \) основана на эмпирических данных о соотношении масс, мы называем это эмпирической формулой соединения. Чтобы определить молекулярную формулу , нам нужно определить относительную массу молекулы соединения, то есть молекулярную массу. Один из способов сделать это основан на законе объединения объемов, гипотезе Авогадро и законе об идеальном газе .Однако для иллюстрации, если бы мы обнаружили, что относительная масса одной молекулы соединения равна 60,0, мы могли бы заключить, что молекулярная формула имеет вид \ (\ ce {C_2O_2H_4} \).

Обзор и вопросы для обсуждения

Укажите закон объединения объемов и приведите пример вашей собственной конструкции, демонстрирующей этот закон.

Объясните, как закон объединения объемов в сочетании с атомно-молекулярной теорией напрямую приводит к гипотезе Авогадро о том, что равные объемы газа при одинаковых температурах и давлении содержат одинаковое количество частиц.

Используйте гипотезу Авогадро, чтобы продемонстрировать, что молекулы газообразного кислорода не могут быть одноатомными.

Плотность водяного пара при комнатной температуре и атмосферном давлении равна \ (0,737 \: \ frac {\ text {g}} {\ text {L}} \). Соединение A представляет собой \ (80,0 \% \) углерод по массе и \ (20,0 \% \) водород. Соединение B представляет собой \ (83,3 \% \) углерод по массе и \ (16,7 \% \) водород. Плотность газообразного Соединения A составляет \ (1.227 \: \ frac {\ text {g}} {\ text {L}} \), а плотность Соединения B составляет \ (2.948 \: \ frac {\ text {g }} {\ text {L}} \).Покажите, как эти данные можно использовать для определения молярных масс соединений A и B, предполагая, что вода имеет молекулярную массу 18.

Из приведенных выше результатов определите массу углерода в молекуле соединения A и в молекуле соединения B. Объясните, как эти результаты показывают, что атом углерода имеет атомную массу 12.

Объясните полезность вычисления количества молей в образце вещества.

Объясните, как мы можем сделать вывод, что \ (28 \: \ text {g} \) газообразного азота \ (\ left (\ ce {N_2} \ right) \) содержит ровно столько же молекул, сколько \ (32 \: \ text {g} \) газообразного кислорода \ (\ left (\ ce {O_2} \ right) \), хотя мы не можем сосчитать это число.

Авторы и авторство

Определение способа вычисления относительной атомной массы элемента Определение относительной изотопной массы gcse Chemistry Calculations igcse O Level revision notes

1. Объясняя и как рассчитать относительную атомную массу RAM или A r из элемент

а) Введение — определение относительной атомной массы — шкала углерода-12

  • Каждый атом имеет свою уникальную относительную атомную масса (RAM) на основе стандартного сравнения или относительной шкалы е.грамм. в прошлом он был основан на водороде H = 1 а.е.м. и кислороде O = 16 а.е.м. (а.е.м. = относительная атомная единица массы).
    • Относительная атомная масса элемента учитывается учитывать различные массы изотопов этого элемента и обилие изотопов в естественном элементе (имеется в виду процент каждого присутствующего изотопа).
    • Относительная атомная масса определена и объяснена ниже, и примеры того, как рассчитать его по данным.
  • Относительный атомный Масштаб массы теперь основан на изотопе углерода , а именно углерода-12 , символ нуклида , которому в соответствии с международным соглашением присвоено произвольное значение 12,0000 а.е.м.
    • Единица «аму» теперь заменяется на нижний регистр u , где u — символ единой атомной массы Блок .
      • Следовательно, один атом углерода, изотопная масса 12, равно 12 u, или,
      • 1 u = 1 / 12 th масса одного атома изотопа углерода-12.
    • Обратите внимание, что для стандартных обозначений нуклидов , верх левое число — это массовое число ( 12 ), а нижнее левое число — это атомное / протонное число ( 6 ).
  • Поскольку относительная атомная масса элемента теперь равна на основе изотопа углерода-12 теперь его можно определить как …
    • относительная атомная масса равна средней массе всех атомов в элементе по сравнению с 1 / 12 -я масса атома углерода-12 (изотоп углерода-12).
    • Примеры показаны в Периодической таблице диаграмма выше.
    • Примечание
      • (i) Из-за наличия нейтронов в ядре, относительная масса атома обычно не менее двух число атомов / протонов, потому что нейтронов обычно больше, чем протоны в ядре (масса протона = 1, нейтрона = 1). Просто отсканируйте таблица Менделеева и исследуйте пары чисел.
        • Вы также должны заметить, что обычно говоря о численной разнице между атомным / протонным числом и относительная атомная масса имеет тенденцию к увеличению с увеличением атомного номера.
        • Это имеет последствия для ядерный стабильность.
      • (ii) Для многих расчетов в целях, относительные атомные массы обычно цитируются и используются при этом академический уровень (уровень GCSE / IGCSE / O) до нуля или одного десятичного знака, например.
        • водород H = 1 ,008 или ~ 1 ; кальций Ca = 40,08 или ~ 40 .0; хлор Cl = 35,45 ~ 35,5 , медь Cu = 63.55 или ~ 63,5 / 64 , серебро Ag = 107,9 или ~ 108 пр.
      • На продвинутом уровне значения относительных атомные массы могут быть указаны с точностью до одного или двух десятичных знаков.
        • Многие атомные массы известны с точностью четырех знаков после запятой, но для некоторых элементов изотопный состав меняется в зависимости от минералогического источника, поэтому четыре десятичных знака не обязательно точнее!
      • Обратите внимание, что в случае углерода есть три изотопы углерода-12 12 C самые распространенные и небольшие количества углерода-13 13 C и углерода-14 14 C.Среднее расчетная масса атомов по сравнению с углеродом 12 составляет 12,01 , но для для большинства целей на довузовском уровне 12.0 является достаточной точностью.
  • При использовании символа A r для RAM, следует иметь в виду, что буква A сама по себе обычно означает массовое число того или иного изотопа. а аму — это сокращение от атомных единиц массы .
  • Однако имеется осложнений, связанных с изотопами и очень точные атомные массы никогда не являются целыми числами.
  • Изотопы — атомы одного элемента с разными массы за счет разного количества нейтронов.
    • Очень точная шкала относительной атомной массы основан на конкретном изотопе углерода , углерода-12, 12 C = 12,0000 единиц точно, для большинства целей C = 12 используется для простоты .
    • Для пример водород-1, водород-2 и водород-3, являются обозначение нуклидов для трех изотопов водорода, хотя подавляющее большинство атомов водорода имеют масса 1.
    • Когда их точные изотопные массы и их процентное содержание равны с учетом средней точной относительной массы для водорода = 1.008, но для большинства целей достаточно H = 1!
    • Видеть также GCSE / IGCSE / AS Atomic Structure Notes
  • Следовательно, более строгое определение относительная атомная масса (A r ) состоит в том, что она равна средней массе всех изотопные атомы, присутствующие в элементе, по сравнению с 1 / 12 th масса атома углерода-12.
    • И, относительная изотопная масса углерода-12 равна присвоено числовое значение 12.0000 .
    • Итак, при вычислении относительной атомной массы вы должен учитывать разные изотопные массы одних и тех же элементов, но и их% изобилие в элементе.
    • Следовательно, вам необходимо знать процент (%) каждого изотопа элемента, чтобы точно вычислить относительную атомную массу элемента.
    • Для приблизительных расчетов относительной атомной массы, вы можете просто использовать массовые числа изотопов, которые очевидно, все целые числа (‘целые числа’!), например. в двух расчетах ниже.
    • С точностью до целого числа, изотопный масса = массовое число для определенного изотопа.
    • Если элемент имеет только один изотоп, относительный атомная масса = относительная масса этого изотопа.
      • Хорошим примером является фтор.
      • Весь фтор атомы имеют массу 19 ( 19 F ), поэтому его относительная атомная масса 19 и никаких расчетов не требуется.

Выше представлена ​​типичная таблица Менделеева, используемая в спецификациях по науке и химии GCSE

.

и я «обычно» использовал эти значения в своих расчетах на примере, чтобы охватить большинство учебные программы


ВЕРХ СТРАНИЦЫ и субиндекс


(б) Примеры расчетов относительной атомной массы для студентов химии уровня GCSE 9-1 / IGCSE / AS / A

Как рассчитать относительную атомную массу?

Ты может рассчитать относительную атомную массу по изотопному содержанию

  • Для точных химических расчетов относительных атомных Следует использовать массу, а не индивидуальное массовое число.
    • Следовательно, относительная атомная масса учитывает все различные «стабильные» изотопы элемента, которые присутствуют в природе.
    • Относительная атомная масса равна средней массе и равна довольно легко рассчитывается из процентного состава (% содержания).
    • Присутствие изотопов объясняет, почему некоторые относительные атомные массы даже не близки к целому числу.
    • Некоторые относительные атомные массы являются почти целыми числами из-за совпадения% изотопов, другие из-за того, что один изотоп может доминировать состав, содержащий лишь крошечные количества более легких или тяжелых изотопов.
  • Пример 1.1. Расчет относительной атомной массы брома. и
    • бром состоит из два изотопа, 50% 79 Br и 50% 81 Br , рассчитайте A r брома от массовых чисел (цифры вверху слева).
    • Думайте о расчетах в терминах «100 атомов»
    • A r = [(50 x 79) + (50 х 81)] / 100 = 80
    • Итак, относительная атомная масса бром 80 или RAM или A r (Br) = 80
    • Обратите внимание на полную показанную работу. Да, хорошо, вы можете делать это в уме, НО многие студенты игнорируют% и просто усредните все заданные изотопные массы (массовые числа), в этом случае бром-79 и бром-81.
    • Элемент бром — единственный известный мне случай, когда усредняется изотопные массы действительно работают! так что будьте осторожны!
  • Пример 1.2. Расчет относительной атомной массы хлора. на основе и изотопы
    • Хлор состоит из два изотопа, 75% хлора-35 и 25% хлора-37 , поэтому использование эти два массовых числа…
    • … снова подумайте о данных, основанных на 100 атомов, поэтому 75 имеют массу 35 и 25 атомов имеют массу 37.
    • Средняя масса = [(75 x 35) + (25 x 37)] / 100 = 35,5
    • Итак, относительная атомная масса хлор 35,5 или RAM или A r (Cl) = 35,5
    • Примечание: 35 Cl и 37 Cl являются наиболее распространенными изотопами хлора, но там — крошечные проценты других изотопов хлора, которые обычно игнорируется на уровне GCSE / IGCSE и Advanced GCE AS / A2.
  • Пример 1.3: Расчет относительной атомной масса меди от ее изотопного состава (изотопное содержание)
    • Медь природного происхождения состоит на 69,2% из меди-63 ( 63 Cu ) и 30,8% меди-65 ( 65 Cu )
    • Все еще мыслим в терминах 100 атомов и не складывайся выкл. десятичными дробями, он все равно работает правильно, потому что 69.2 + 30,8 = 100!
    • средняя масса = относительная атомная масса меди = {(63 x 69,2) + (65 x 30,8)} / 100 = 63,6
  • Пример 1.4: Атомы серебра состоят на 51,4% из изотоп 107 Ag и 48,6% изотопа 109 Ag
    • Рассчитайте относительную атомную массу серебра.
    • (51.4 х 107) + (48,6 х 109) 5499,8 + 5297,4
      A r (Ag) = ————————————— = ————————— = 108,0
      100 100
    • Относительная атомная масса серебра 108.0 (до 1 десятичный знак)
  • Пример 1.5: Атомы европия состоят из 47,8% Eu-151 и 52,2% Eu-153
    • Рассчитайте относительную атомную массу европия.
    • (47,8 x 151) + (52,2 х 153) 7217,8 + 7986,6
      A r (Eu) = ————————————— = ————————— = 152.0
      100 100
    • Относительная атомная масса европия 152,0 (к 1 десятичный знак)
  • Пример 1.6: Атомы элемента кремния состоят из 92,2% кремния-28, 4,7% кремния-29 и 3,1% кремния-30.
    • Вычислите относительную атомную массу кремния.
    • (92,2 х 28) + (4,7 х 29) + (3,1 х 30) 2581,6 + 136,3 + 93,0
      A r (Si) = ————————————————— = ——————————— = 28.1
      100 100
    • Относительная атомная масса кремния 28,1 (к 1 десятичный знак или 3 значащих цифры)
  • См. Ниже и масс-спектрометр и изотопный анализ на GCSE-Advanced A Level (базовый) Atomic Structure Notes, с дальнейшими расчет относительной атомной массы.


(c) Примеры только для студентов, изучающих химию на продвинутом уровне A

Как рассчитать относительную атомную массу с точным относительным изотопные массы

На основе данных современных высокоточных масс-спектрометров

(a) Очень точный расчет относительной атомной массы (необходимо знать и определить, что такое относительная изотопная масса )

Родственник изотопная масса определяется как точная масса одного изотопа элемент по сравнению с 1 / 12 th масса атом углерода-12 e.грамм. точная относительная изотопная масса кобальта-5 это 58.9332

Это определение относительной изотопной массы: полностью отличается от определения относительной атомной массы, за исключением оба основаны на одном и том же международном стандарте атомной массы, т. е. 1 единица (1 ед.) = 1/12 массы изотопа углерода-12 ( 12 C ).

Если бы мы переделали расчет относительная атомная масса хлора (пример 1.1 выше), что вполне подходит для целей GCSE (и, возможно, уровня A), но точнее в A уровень, сделаем ….

хлор 75,77% 35 Cl изотопная масса 34,9689 и 24,23% 37 Cl изотопной массы 36,9658

так A r (Cl) = [(75,77 x 34,9689) + (24,23 x 36,9658)] / 100

= 35,4527 (но в довузовских расчетах обычно нормально 35,5!)

См. Также Масс-спектрометр и изотопный анализ в примечаниях к атомной структуре уровня GCSE / A с дальнейшими расчетами RAM.

(б) Расчеты% состава изотопов

Можно сделать обратное вычисления относительной атомной массы, если вы знаете A r и какие изотопы присутствуют.

Это включает в себя немного арифметическая алгебра.

Бора A r 10.81 и состоит всего из двух изотопов, бора-10 и бора-11

.

Относительная атомная масса бор был точно получен в прошлом химическим анализом реагирующих массы, но сейчас масс-спектрометры могут сортировать из всех присутствующих изотопов и их относительного содержания.

Если вы положите X =% бора 10, тогда 100-Х равно% бора-11

Следовательно, A r (B) = ( X x 10) + [(100– X ) x 11)] / 100 = 10,81

так, 10 X -11 X +1100 = 100 х 10,81

-X + 1100 = 1081, 1100 — 1081 = X (сменить сторону поменять знак!)

, следовательно, X = 19

так естественно встречающийся бор состоит из 19% 10 B и 81% 11 B

(г. Справочники фактически цитируют 18.7 и 81,3, но мы не использовали очень точный относительные изотопные массы, указанные выше!)


ВЕРХ СТРАНИЦЫ и субиндекс


Примечания к редакции о том, как определить относительную атомную массу и как рассчитать относительную атомную массу из процентное содержание изотопов, помогает в пересмотре AQA уровня A, Edexcel, OCR 21st Centre, Gateway science GCSE 9-1, экзамен по химии


На других страницах на Атомная структура и Относительная формула Масса

Тесты для самооценки на относительную атомную массу

введите ответ Викторина или множественный выбор ВИКТОРИНА


ПРИЛОЖЕНИЕ 1.Типичная таблица Менделеева, используемая на предвузовских экзаменах

Выше представлена ​​типичная таблица Менделеева, используемая в спецификациях по науке и химии GCSE в делать химические расчеты, и я «обычно» использовал эти значения в своих расчетах на примере, чтобы охватить большинство учебные программы

Примечания к редакции о том, как определить относительную атомную массу и как рассчитать относительную атомную массу из процентное содержание изотопов, помогает в пересмотре AQA уровня A, Edexcel, OCR 21st Centre, Gateway science GCSE 9-1, экзамен по химии


ВЕРХ СТРАНИЦЫ и субиндекс


(г) ПРИЛОЖЕНИЕ 2.Таблица относительных атомных масс для элементов с 1 по 92

Примечания: (i) Список относительных атомных масс находится в в алфавитном порядке по названию элемента вместе с химическим символом и протон / атомный номер.

(ii) Относительные атомные массы указаны с точностью до второго знака после запятой. места, хотя важно знать, что ценности в доуниверситетском результаты экзаменов можно округлить до ближайшего целого числа или до одного десятичного знака.

(iii) Трансурановые элементы были исключены, поскольку их изотопный состав меняется в зависимости от источника e.грамм. циклотрон, ядерный реактор и т. д. И все их изотопы очень радиоактивны, и большинство из них очень нестабилен (поэтому ваша относительная атомная масса все время меняется!)

(iv) * радиоактивный, массовое число наиболее стабильного изотопа цитируется

г.
Химический символ
Имя элемента
Атомный №Z
Относительная атомная масса
Ac Актиний 89 227,03
Al Алюминий 13 26,98
Сб Сурьма 51 121.75
Ar Аргон 18 39,95
As Мышьяк 33 74,92
На Астатин 85 210 *
Ba Барий 56 137.33
Be Бериллий 4 9,01
Би Висмут 83 208,98
B Бор 5 10.81
Br Бром 35 79,90
Cd Кадмий 48 112,41
CS Цезий 55 132.91
Ca Кальций 20 40,08
C Углерод 6 12.01
CE Церий 58 140.12
Cl Хлор 17 35,45
Cr Хром 24 52,00
Co Кобальт 27 58.93
Cu Медь 29 63,55
Dy Диспрозий 66 162,50
Er Эрбий 68 167.26
Eu Европий 63 151,97
F Фтор 9 19.00
Fr Франций 87 223 *
Gd Гадолиний 64 157.25
Ga Галлий 31 69,72
Ge Германий 32 72,60
Au Золото 79 196.97
Hf Гафний 72 178,49
Он Гелий 2 4,00
Ho Гольмий 67 164.93
H Водород 1 1.01
В Индий 49 114,82
Я Йод 53 126.90
Ir Иридий 77 192,22
Fe Утюг 26 55,85
Кр Криптон 36 83.80
La Лантан 57 138,91
Pb Свинец 82 207,20
Ли Литий 3 6.94
Lu Лютеций 71 174,97
Mg Магний 12 24,31
Mn Марганец 25 54.94
Hg Меркурий 80 200,59
руб.
Химический символ
Имя элемента
Атомный №Z
Относительная атомная масса
Пн Молибден 42 95,94
Nd Неодим 60 144,24
Ne Неон 10 20.18
Ni Никель 28 58,69
Nb Ниобий 41 92,91
Азот 7 14.01
Os Осмий 76 190,20
O Кислород 8 16.00
Pd Палладий 46 106.42
П Фосфор 15 30,97
Pt Платина 78 195.08
Po Полоний 84 209 *
K Калий 19 39.10
Pr Празеодим 59 140,91
PM Прометий 61 145 *
Pa Протактиний 91 231.04
Ra Радий 88 226,03
Rn Радон 86 222 *
Re Рений 75 186.21
Rh Родий 45 102,91
Рубидий 37 85,47
Ru Рутений 44 101.07
Sm Самарий 62 150,36
SC Скандий 21 44,96
Se Селен 34 78.96
Si Кремний 14 28.09
Ag Серебро 47 107,87
Na Натрий 11 23.00
Sr Стронций 38 87,62
S Сера 16 32,07
Ta Тантал 73 180.95
TC Технеций 43 98,91
Te Теллур 52 127,60
Tb Тербий 65 158.93
Tl Таллий 81 204,38
Чт Торий 90 232,04
ТМ Тулий 69 168.93
Sn Олово 50 118,71
Ti Титан 22 47,88
W Вольфрам 74 183.85
U Уран 92 238,03
V Ванадий 23 50,94
Xe Ксенон 54 131.29
Yb Иттербий 70 173,04
Y Иттрий 39 88,91
Zn Цинк 30 65.39
Zr Цирконий 40 91,22

Примечания к редакции о том, как определить относительную атомную массу и как рассчитать относительную атомную массу из процентное содержание изотопов, помогает в пересмотре AQA уровня A, Edexcel, OCR 21st Centre, Gateway science GCSE 9-1, экзамены по химии расчет относительной атомной массы как рассчитать относительную атомную массу брома из процентного содержания изотопов, как рассчитать относительная атомная масса хлора от процентного содержания изотопов, как рассчитать процентное содержание изотопов в элементе с учетом его относительная атомная масса

ВЕРХ СТРАНИЦЫ и субиндекс


ДРУГИЕ РАСЧЕТНЫЕ СТРАНИЦЫ

  1. Что такое относительная атомная масса ?, относительная изотопная масса и расчет относительной атомной массы (эта страница)

  2. Расчет относительной формула / молекулярная масса соединения или молекулы элемента

  3. Закон сохранения массы и простые вычисления реагирующей массы

  4. Состав по процентной массе элементов в соединении

  5. Эмпирическая формула и формула массы соединения из реагирующих масс (легкий старт, без родинок)

  6. Расчет соотношения реагирующих масс реагентов и продуктов из уравнений (Не используя моль) и краткое упоминание фактического процентного выхода и теоретического выхода, атомная экономика и определение массы по формуле

  7. Введение в моли: связь между молями, массой и формульной массой — основа расчета молярных соотношений реагирующих веществ. (относительно реагирующих масс и формулы масса)

  8. С помощью моль, чтобы вычислить эмпирическую формулу и вывести молекулярную формулу соединения / молекулы (начиная с реагирующих масс или% состава)

  9. Моли и молярный объем газа, закон Авогадро

  10. Объем реагирующего газа отношения, закон Авогадро и закон Гей-Люссака (соотношение газообразных реагенты-продукты)

  11. Молярность, объемы и раствор концентрации (и схемы аппаратов)

  12. Как сделать кислотно-щелочной расчеты титрования, схемы аппаратов, подробности процедур

  13. Расчет продуктов электролиза (отрицательный катод и положительный анод)

  14. Прочие расчеты е.грамм. % чистоты,% процентного содержания и теоретический выход, разбавление растворов (и схемы аппаратов), кристаллизационная вода, количество реагентов требуется, атом эконом

  15. Передача энергии при физических / химических изменениях, экзотермические / эндотермические реакции

  16. Расчеты по газу с учетом отношений PVT, Лоулз Бойля и Чарльза

  17. Расчеты радиоактивности и периода полураспада, включая датирующие материалы



Ключевые слова и фразы: Какова относительная атомная масса элемент? На какой шкале основана относительная атомная масса? Какая формула вычислить относительную атомную массу элемента? Количественная химия расчеты онлайн Помощь для решения проблем в расчетах относительной атомной массы.Определения относительных атомных масса и относительная изотопная масса (только учащиеся уровня A) Практикуйте повторные вопросы на определение относительной атомной массы из изотопного состава (% изотопов, Студенты уровня A узнают об очень точных данных масс-спектрометра). Что такое относительное атомарное масса? Как рассчитать относительную атомную массу элемента. Что такое стандартная единица массы? Относительная атомная масса поясняется ниже со ссылкой на углерод-12. атомная шкала масс и актуальность изотопов и единой буквы объяснена атомная единица массы.Подробные примеры полностью объяснен метод расчета относительной атомной массы из изотопного состава со ссылкой на определение относительной атомной массы сложный. Для учащихся уровня A: как определять и использовать относительные изотопы массы для расчета относительной атомной массы. Эти примечания по определению, объяснению и вычислению относительных атомная масса и определяющая относительная изотопная масса равны разработан в соответствии с высочайшими стандартами требуемых знаний и понимания для студентов / школьников, изучающих химию GCSE, химию IGCSE, O Уровень химии, курсы естествознания KS4 и курсы химии уровня A.


как определить относительную атомную массу Расчеты Редакция KS4 Наука пересматривает, как сделать относительную атомную массу расчеты Дополнительные Тройная награда в области естественных наук Курсы по отдельным наукам помогают научиться относительной редакция учебника по расчетам атомной массы Уровень GCSE / IGCSE / O Химия, как проводить расчеты относительной атомной массы Информационные заметки для изучения для пересмотра для AQA GCSE Наука, как проводить расчеты относительной атомной массы, Edexcel GCSE Science / IGCSE Chemistry как выполнять вычисления относительной атомной массы и OCR 21st Century Science, OCR Gateway Наука как проводить расчеты относительной атомной массы WJEC gcse science chemistry как проводить расчеты относительной атомной массы CEA / CEA gcse science chemistry O Уровень химии (пересмотреть курсы, равные 8 классу США, 9 класс 10 классу как делать расчет относительной атомной массы) Уровень Примечания к редакции для продвинутого вспомогательного уровня GCE, как определить относительную атомную массу Расчеты AS Продвинутый Уровень A2 IB Пересмотр того, как рассчитать относительную атомную массу расчеты AQA GCE Chemistry OCR GCE Chemistry как проводить расчеты относительной атомной массы Edexcel GCE Chemistry Salters Chemistry как сделать расчет относительной атомной массы CIE Химия как проводить расчеты относительной атомной массы, WJEC GCE AS A2 Chemistry как проводить расчеты относительной атомной массы, CCEA / CEA GCE AS A2 Chemistry revision как проводить курсы по расчету относительной атомной массы для студентов доуниверситетских учебных заведений (соответствует 11-му и 12-му классам в США и уровню AP Honors / Honors как делать пересмотренное руководство по расчетам относительной атомной массы о том, как делать относительные атомные массовые расчеты относительная атомная масса 1 водорода H относительная атомная масса 2 гелия He относительная атомная масса 3 лития Относительная атомная масса Li 4 Бериллия Be относительная атомная масса масса 5 Бор B относительная атомная масса 6 углерода C относительная атомная масса 7 азота N относительная атомная масса 8 Кислород O относительная атомная масса 9 Относительная атомная масса фтора F 10 относительных Neon Ne атомная масса 11 Натрий Na относительная атомная масса 12 Относительная атомная масса магния Mg 13 Al относительная атомная масса 14 кремния Si относительная атомная масса 15 фосфора P относительная атомная масса 16 Sulphur S относительная атомная масса 17 хлора Относительная атомная масса Cl для 18-го аргона Относительная атомная масса Ar 19 Калий K относительная атомная масса 20 Относительная атомная масса кальция Ca относительно 21 Scandium Sc атомная масса 22 титана Ti относительная атомная масса 23 ванадия V относительная атомная масса 24 хрома Cr относительная атомная масса 25 марганца Относительная атомная масса Mn для относительной атомной массы железа Fe 26 27 Cobalt Co относительная атомная масса 28 Никель Ni относительная атомная масса 29 Copper Cu относительная атомная масса 30 Цинк Zn относительная атомная масса 31 Галлий Ga относительная атомная масса 32 германия Ge относительно атомная масса 33 Arsenium As относительная атомная масса 34 Selenium Se относительная атомная масса 35 Бром Br относительная атомная масса 36 Криптон Kr относительная атомная масса 37 Рубидий Rb относительная атомная масса масса 38 Стронций Sr относительная атомная масса 39 Относительная атомная масса иттрия Y 40 циркония Zr относительная атомная масса 41 ниобия ниобия относительная атомная масса 42 молибдена молибдена относительная атомная масса 43 Technetium Tc относительная атомная масса 44 Рутений Ru относительная атомная масса 45 Родий Rh относительная атомная масса 46 Palladium Pd относительная атомная масса 47 Silver Ag относительная атомная масса 48 кадмия Cd относительная атомная масса 49 индия In относительная атомная масса 50 олова Sn относительная атомная масса 51 относительный атомный масса сурьмы Sb относительная атомная масса 52 Относительная атомная масса теллура Te 53 относительная атомная масса йода I. атомная масса 54 Xenon Xe относительная атомная масса 55 Относительная атомная масса цезия Cs 56 относительная атомная масса бария Ba атомная масса 57 лантана La относительная атомная масса 58 Церий Ce относительная атомная масса 59 празеодима Pr относительная атомная масса 60 неодима Nd относительная атомная масса 61 прометия Pm относительная атомная масса 62 Samarium Sm относительная атомная масса 63 европия Eu относительная атомная масса 64 гадолиния Gd относительная атомная масса 65 тербия Tb относительная атомная масса 66 Dysprosium Dy относительная атомная масса 67 гольмия Ho относительная атомная масса 68 Erbium Er относительная атомная масса 69 Thulium Tm относительная атомная масса 70 Ytterbium Yb относительная атомная масса 71 лютеция Lu относительная атомная масса 72 гафния Hf относительная атомная масса 73 тантала Ta относительная атомная масса 74 вольфрама W относительная атомная масса 75 рений Re относительная атомная масса осмия 76 Os относительная атомная масса 77 иридия Ir относительная атомная масса 78 Platinum Pt относительная атомная масса 79 золота Au относительная атомная масса 80 ртути Hg относительная атомная масса 81 таллия Tl относительная атомная масса 82 Свинец Pb относительная атомная масса 83 висмута Bi относительная атомная масса 84 Полоний Po 85 Астатин При относительной атомной массе 86 Относительная атомная масса радона Rn 87 Франций Fr относительная атомная масса 88 Относительная атомная масса радия Ra 89 Актиний Ас относительная атомная масса 90 Относительная атомная масса тория Th 91 Протактиний Па относительная атомная масса 92 Уран U gcse: редакция химии бесплатные подробные заметки по расчету относительной атомной массы элемент, помогающий пересмотреть химию igcse Заметки о пересмотре химии igcse по расчету относительных атомная масса элемента O уровень химия подробные примечания по вычислению относительных атомных масса элемента, чтобы помочь пересмотреть gcse бесплатные подробные заметки по химии по расчету относительных атомных масса элемента, чтобы помочь пересмотреть уровень O бесплатный онлайн-сайт по химии, чтобы помочь пересмотреть расчет относительная атомная масса элемента для gcse бесплатный веб-сайт по химии, чтобы помочь пересмотреть расчет относительной атомной массы элемента для Бесплатный онлайн-сайт по химии igcse, который поможет пересмотреть уровень O вычисление относительной атомной массы элемента химия как добиться успеха в вопросах по расчет относительной атомной массы элемента для gcse химия как добиться успеха в igcse химия как добиться успеха по химии уровня O хороший сайт для бесплатных вопросов по вычисление относительной атомной массы элемента, чтобы помочь сдать вопросы по химии gcse расчет относительной атомной массы элемента хороший сайт бесплатно помогите пройти igcse химия с доработкой примечания по расчету относительной атомной массы элемента хороший сайт для бесплатной помощи для прохождения уровня O химия



ВЕРХ СТРАНИЦЫ и субиндекс

Учебник по химии относительной атомной массы

Ключевые концепции

  • Относительная атомная масса элемента также известна как относительный атомный вес элемента или атомный вес элемента.
  • Относительная атомная масса часто обозначается аббревиатурой r.a.m.
  • Относительная атомная масса элемента (его атомный вес) указана в Периодической таблице.
  • Относительная атомная масса элемента — это средневзвешенная масса изотопов в природном элементе относительно массы атома изотопа углерода-12, которая принята равной точно 12.
  • Атомная единица массы (u) определяется как эквивалент массы 1 / 12 массы одного атома углерода-12.

    1 u = 1,66 × 10 -27 кг

  • Мы можем оценить относительную атомную массу (атомный вес) элемента E с встречающимися в природе изотопами a E, b E, c E и т. Д. И с соответствующими содержаниями A % , B %, C % и т. Д.,
    относительная атомная масса (r.a.m.) = ( А
    100
    × а ) + ( B
    100
    × б ) + ( С
    100
    × c ) + и т. Д.
  • Учитывая относительную атомную массу (г.утра) элемента и предполагаемую массу каждого из его изотопов, мы можем затем оценить относительное содержание каждого изотопа:
    let x = процентное содержание изотопа-а
    и 100 — x = процентное содержание изотопа-b
    тогда пусть r.a.m = относительная атомная масса элемента:
    п.м. = ( x
    100
    × масса изотопа-а) + ( 100 — x
    100
    × масса изотопа-b)

    и решите для x

  • Обратите внимание, что мы можем измерить массу каждого изотопа и его содержание с помощью масс-спектроскопии.

Пожалуйста, не блокируйте рекламу на этом сайте.
Нет рекламы = нам нет денег = нет бесплатных вещей для вас!

Обилие изотопов естественного происхождения

Большинство элементов встречаются в природе в виде смеси различных изотопов.

Например, элемент углерод существует в природе как смесь различных изотопов: стабильных 1 атомов углерода-12 и углерода-13.

Если вы возьмете образец атомов углерода, например сажу из дымохода или кусок угля, вы обнаружите, что большинство атомов углерода являются изотопом углерода-12, и только некоторые из них будут изотопом углерода-13. изотоп.
Мы называем это количество каждого изотопа, обнаруженного в естественном элементе, его распространенностью, или, если быть более точным, его изотопным содержанием.
Содержание изотопа углерода-12 в естественном массовом углероде составляет 98,90%, в то время как содержание изотопа углерода-13 в природе составляет 1,10%.
Это означает, что если вы возьмете кусок угля из природы, 98,90% углерода в угле будут атомами изотопа углерода-12, в то время как только 1,10% будут атомами изотопа углерода-13.

В таблице ниже приведены изотопные содержания некоторых элементов на Земле:

Элемент Изотоп Обилие (%) Элемент Изотоп Обилие (%)
водород 1 H 99,99 натрий 23 Na 100.00
2 H 0,01 магний 24 мг 78,90
гелий 3 He 0,0001 25 мг 10,00
4 He 99,9999 26 мг 11.10
литий 6 Li 7,42 алюминий 27 Al 100,00
7 Li 92,58 кремний 28 Si 92,23
бериллий 9 Be 100.00 29 Si 4,67
бор 10 В 19,80 30 Si 3,10
11 B 80,20 фосфор 31 п 100
углерод 12 С 98.90 сера 32 S 95,02
13 С 1,10 33 S 0,75
азот 14 99,63 34 S 4,21
15 N 0.37 36 S 0,02
кислород 16 O 99,76 хлор 35 Класс 75,77
17 O 0,038 37 Класс 24.23
18 O 0,20 аргон 36 Ар 0,34
фтор 19 ф 100,00 38 Ар 0,063
неон 20 Ne 90.60 40 Ар 99,60
21 Ne 0,26
22 Ne 9,20

Вы можете найти более полный список изотопных содержаний внизу этой страницы.

Оценка изотопной массы

Относительная атомная масса атома углерода-12 определяется как 12,00
Относительная атомная масса атома углерода-13 оказалась в 1,08333 раза больше массы атома углерода-12, то есть 1,083 × 12 = 13,00.

Мы можем оценить массу любого изотопа элемента, его изотопную массу, используя его массовое число (A).

Массовое число (A) изотопа говорит нам, сколько протонов и нейтронов находится в ядре атома этого изотопа.
Нуклон — это термин, используемый для описания как протонов, так и нейтронов.
Итак, массовое число (A) сообщает нам количество нуклонов в ядре атома изотопа элемента.

Например:

  • ядро ​​атома изотопа углерода-12 содержит 12 нуклонов
  • ядро ​​атома изотопа углерода-13 содержит 13 нуклонов

Масса протона почти такая же, как масса нейтрона.
Масса протона составляет около 1 u (1 атомная единица массы), поэтому масса нейтрона также около 1 u.
Масса электрона настолько мала по сравнению с массой протона или нейтрона, что ею можно пренебречь при оценке массы изотопа элемента. 2

Мы можем оценить массу атома изотопа элемента, сложив массу его нуклонов: 3

изотопическая масса = количество нуклонов × масса нуклона

изотопная масса = количество нуклонов × 1 u

Например:

  • изотопная масса атома углерода-12: 12 нуклонов × 1 u / нуклонов = 12 u
  • изотопная масса атома углерода-13: 13 нуклонов × 1 ед. / нуклонов = 13 ед.

Изотопные массы можно измерить с помощью масс-спектроскопии.Вы найдете обсуждение расчета относительной атомной массы (атомного веса) с использованием этих измеренных изотопных масс в учебном пособии по масс-спектроскопии.

Расчет относительной атомной массы (атомного веса) элемента

Относительная атомная масса элемента — это средневзвешенная масса изотопов в природном элементе относительно массы атома изотопа углерода-12, которая принимается равной точно 12.

Что такое «средневзвешенное значение»?

Сначала давайте посмотрим, какой будет «средний вес» углерода:

Масса изотопа углерода-12 составляет 12 ед.

Масса изотопа углерода-13 13 и

Итак, мы можем рассчитать среднюю массу (средний вес) углерода как:

средний вес = 12 + 13
2
= 12.5 u

Если мы посмотрим на атомный вес углерода в Периодической таблице, мы обнаружим, что это 12,01 НЕ 12,5.

Это связано с тем, что большинство атомов углерода, встречающихся в природе, являются атомами изотопа углерода-12, в то время как очень немногие из атомов относятся к изотопу углерода-13.
Нам необходимо учитывать это при вычислении нашего «среднего», и когда мы делаем это, мы называем результат «средневзвешенным».

Нам нужно знать количество каждого изотопа, то есть нам нужно знать, какая часть массы (веса) природного углерода в объеме приходится на каждый из изотопов (углерод-12 и углерод-13).
Из таблицы в разделе выше мы находим, что изотопное содержание углерода-12 составляет 98,90%, а изотопное содержание углерода-13 составляет 1,10%.

Это означает, что если бы у меня было 100 атомов углерода в объеме (например, в угле или сажи), то:

  • 98,90% из 100 атомов углерода составляют атомы углерода-12 с массой 12 ед.
    То есть 98,90 атомов углерода имеют массу 12 u
  • 1,10% из 100 атомов углерода составляют атомы углерода-13 с массой 13 ед.
    То есть 1.10 атомов углерода имеют массу 13 ед.

Итак, общая масса 100 встречающихся в природе атомов углерода равна:

общая масса
из 100 атомов
= масса всех
атомов углерода-12
+ масса всех
атомов углерода-13
= 98.90 × 12 + 1,10 × 13
= 1186,8 + 14,3
= 1201,1

Итак, масса 100 встречающихся в природе атомов углерода равна 1201.1 ты
Следовательно, «средневзвешенная масса» 1 атома углерода равна 1201,1 ÷ 100 = 12,011 ед.
Это значение атомной массы углерода согласуется со значением в Периодической таблице.

Обратите внимание, что это НЕ означает, что масса 1 атома углерода равна 12,011 ед.
Это ДЕЙСТВИТЕЛЬНО означает, что если мы возьмем образец встречающегося в природе углерода, мы обнаружим, что средняя масса атома углерода будет 12,011 ед.

Давайте рассмотрим, как мы вычисляли «средневзвешенную» массу объемных атомов углерода:

  • содержание первого изотопа, умноженное на массу этого изотопа, заданную массовым числом
  • содержание второго изотопа, умноженное на массу этого изотопа, заданную массовым числом
  • сложите эти два значения вместе и разделите на 100

В целом, чтобы рассчитать «средневзвешенную» массу элемента, который встречается в природе в виде двух разных изотопов, изотопа 1 и изотопа 2, тогда:

«средневзвешенная»
масса элемента
= (изотопное содержание 1 × массовый изотоп 1) + (изотопное содержание 2 × массовый изотоп 2)
100

Если мы оценим массу каждого изотопа, используя его массовое число (A), то мы можем переписать выражение как:

«средневзвешенная»
масса элемента
= (изотоп содержания 1 × A (изотоп 1) ) + (изотоп содержания 2 × A (изотоп 2) )
100

Или, другими словами:

«средневзвешенная»
масса элемента
= ( Изотоп с содержанием % 1
100
× A (изотоп 1) ) + ( Изотоп с содержанием % 2
100
× A (изотоп 2) )

Расчет изотопного содержания по атомному весу

Периодическая таблица дает нам средневзвешенное значение массы элемента, называемое атомным весом элемента (или относительной атомной массой).

Если мы знаем массу каждого изотопа, составляющего этот природный элемент (оцениваемую его массовым числом), то мы можем вычислить распространенность каждого изотопа в природе.

Мы написали выше общее математическое выражение (математическое уравнение) для расчета «средневзвешенной» массы, также известной как относительная атомная масса или атомный вес, элемента, которая была:

«средневзвешенная»
масса элемента
= (изотоп содержания 1 × A (изотоп 1) ) + (изотоп содержания 2 × A (изотоп 2) )
100

Допустим, я хотел найти содержание (%) каждого изотопа азота.

Азот содержит два природных стабильных изотопа: азот-14 и азот-15.

Итак, подставив их в математическое уравнение выше, я получу:

«средневзвешенная»
масса азота
= (содержание азота-14 × A (азот-14) ) + (содержание азота-15 × A (азот-15) )
100

Я могу найти атомный вес (относительную атомную массу или «средневзвешенную» массу) азота в Периодической таблице:

атомный вес азота 14.01

Я могу оценить массу атома каждого изотопа, используя его массовое число (A):

Масса атома азота-14 = его массовое число = 14 ед.

Масса атома азота-15 = его массовое число = 15 ед.

Я могу подставить эти значения в математическое уравнение выше:

14.01 = (содержание азота-14 × 14) + (содержание азота-15 × 15)
100

Я могу умножить обе части математического уравнения на 100:

100 × 14.01 = 100 × (содержание азота-14 × 14) + (содержание азота-15 × 15)
100
1401 = (содержание азота-14 × 14) + (содержание азота-15 × 15)

Но как я могу решить это уравнение, когда есть 2 неизвестных, содержание азота-14 неизвестно, а содержание азота-15 неизвестно.

Уловка состоит в том, чтобы помнить, что мы говорим о процентах изобилия! Что обозначает:

9000 4% содержание азота-14 +% содержание азота-15 = 100

Или, по-другому:

9000 4% содержание азота-14 = 100% содержание азота-15

Итак, если я позволю процентному содержанию азота-14 быть равным x , тогда:

% содержание азота-14 = x

% содержание азота-15 = 100 — x

Если я подставлю их в математическое уравнение выше, у меня будет только одно неизвестное значение, x :

1401 = (обилие азота-14 × 14) + (обилие азота-15 × 15)
1401 = ( х × 14) + ([100 — x ] × 15)

Чтобы решить для x , сначала сниму скобки:

1401 = ( х × 14) + ([100 — x ] × 15)
1401 = 14 x + (15 × 100) — 15 x
1401 = 14 x + 1500-15 x

Далее я собираю похожие термины, начиная с x :

1401 = 14 x + 1500-15 x
1401 = 14 x -15 x + 1500
1401 = х + 1500

Затем вычтем 1500 из обеих частей уравнения:

1401-1500 = х + 1500-1500
-99 = х

Обратите внимание, что содержание изотопа должно быть положительным числом (а не отрицательным числом), поэтому я делю обе части уравнения на -1, чтобы найти значение x :

99
-1
= x
-1
99 = х

Затем замените это значение на x обратно в выражения, которые мы написали для содержания каждого изотопа:

% содержание азота-14 = x = 99%

% содержание азота-15 = 100 — x = 100 — 99 = 1%

В общем, если элемент встречается в природе в двух изотопных формах, изотопе 1 и изотопе 2, то мы можем оценить процентное содержание каждого изотопа, используя массовое число (A) каждого изотопа, поскольку процентное содержание изотопа 2 равно 100. -% содержание изотопа 1:

100 × атомный вес элемента = (изотоп 1 × A (изотоп 1) ) + ([100 — изотоп 1] × A (изотоп 2) )

лет изотопа 1 = x

100 × атомный вес элемента = ( x × A (изотоп 1) ) + ([100 — x ] × A (изотоп 2) )
100 × атомный вес элемента = x A (изотоп 1) + 100A (изотоп 2) — A (изотоп 2) x
(100 × атомный вес элемента) — (100 × A (изотоп 2) ) = x A (изотоп 1) x A (изотоп 2)
100 × (атомная масса элемента — A (изотоп 2) ) = x (A (изотоп 1) — A (изотоп 2) )
100 × (атомная масса элемента — A (изотоп 2) )
(A (изотоп 1) — A (изотоп 2) )
= х

Итак,

  • % изотопа 1 = x
  • % изотопа 2 = 100 — x

Рабочий пример 1: Расчет атомного веса элемента

Вопрос: Серебро природного происхождения — 51.84% серебра-107 и 48,16% серебра-109.

Вычислите атомный вес серебра.

Решение:

(на основе подхода StoPGoPS к решению проблем.)

  1. Что вас просят сделать?

    Вычислить «средневзвешенную» массу (относительную атомную массу или атомный вес) серебра.

  2. Какие данные (информацию) вы указали в вопросе?

    Извлеките данные из вопроса:

    название изотопа массовое число (А) изотопное содержание (%)
    серебристый-107 107 51.84
    серебро-109 109 48,16
  3. Какая связь между тем, что вы знаете, и тем, что вам нужно выяснить?
    (a) массу изотопа можно оценить, используя его массовое число (A), выраженное в атомных единицах массы, u

    (б)

    «средневзвешенная»
    масса серебра
    = ( % содержание серебра-107
    100
    × А (серебристый-107) ) + ( % содержание серебра-109
    100
    × А (серебристый-109) )
  4. Подставьте значения и решите:
    «средневзвешенная»
    масса серебра
    = ( 51.84
    100
    × 107) + ( 48,16
    100
    × 109)
    = 55,469 + 52,494
    = 107,96
  5. Правдоподобен ли ваш ответ?
    Сделайте приблизительный расчет:
    Поскольку около половины (& приблизительно; 50%) атомов серебра составляют серебро-107, а другая половина (& приблизительно; 50%) — серебро-109, средневзвешенная масса серебра будет находиться примерно посередине между массой серебро-107 (107) и серебро-109 (109):
    атомный вес & ок. ½ × (107 + 109) = 108
    Начиная с нашего расчетного значения 107.96 близко к 108, мы достаточно уверены, что наш ответ правильный.
    Вы также можете посмотреть атомный вес серебра в периодической таблице и узнать, что он равен 107,9.
    Поскольку наше рассчитанное значение 107,96 примерно такое же, как значение в периодической таблице, мы достаточно уверены, что наш ответ правдоподобен.
  6. Изложите свое решение задачи «атомный вес серебра»:

    атомный вес серебра 107.96 u

Рабочий пример 2: Расчет изотопного содержания элемента

Вопрос: Медь состоит из двух изотопов: меди-63 и меди-65.
Его атомный вес (относительная атомная масса) равен 63,62 ед.

Найдите процентное содержание каждого изотопа.

Решение:

(На основе подхода StoPGoPS к решению проблем.)

  1. Что вас просят сделать?

    (a) Рассчитайте процентное содержание меди-63

    (b) Рассчитайте процентное содержание меди-65.

  2. Какие данные (информацию) вы указали в вопросе?

    Извлеките данные из вопроса:

    название изотопа массовое число (А) приблизительная масса изотопа (u)
    медь-63 63 63
    медь-65 65 65

    атомный вес меди = 63.62 u

  3. Какая связь между тем, что вы знаете, и тем, что вам нужно выяснить?

    (а)

    «средневзвешенная»
    масса меди
    = ( % содержание меди-63
    100
    × А (медь-63) ) + ( % содержание меди-65
    100
    × А (медь-65) )

    (б) пусть% содержания меди-63 = x
    и% содержания меди-65 = 100 — x

  4. Подставьте значения в уравнение и решите:

    (а)

    63.62 = ( x
    100
    × 63) + ( 100 — x
    100
    × 65)
    63,62 = 63 x
    100
    + (65 × 100) — 65 x
    100
    63.62 = 63 x
    100
    + 6500-65 x
    100
    63,62 = 63 x + 6500-65 x
    100
    100 × 63.62 = 100 × 63 x + 6500-65 x
    100
    6362 = 63 x + 6500-65 x
    6362 =-2 х + 6500
    6362-6500 =-2 х
    -138 =-2 х
    -138
    -2
    = -2 x
    -2
    69 = х

    (b)% содержания меди-63 = x = 69% 9 2012% содержание меди-65 = 100 — x = 100 — 69 = 31%

  5. Правдоподобен ли ваш ответ?
    Атомный вес меди равен 63.62.
    Поскольку эта масса ближе к массе изотопов меди-63, чем к массе изотопов меди-65, мы знаем, что медь-63 должна быть более распространенной (более 50%).
    Наше рассчитанное значение содержания меди-63 составляет 69%, что превышает 50%, поэтому мы достаточно уверены, что наш ответ правдоподобен.
    Обратите внимание, что процентное содержание обоих изотопов должно составлять в сумме 100%.
    69% + 31% = 100%
    Итак, мы достаточно уверены, что наши расчетные значения содержания каждого изотопа правдоподобны.
  6. Изложите свое решение проблемы «содержание каждого изотопа меди»:

    содержание меди-63 = 69%

    содержание меди-65 = 31%


Сноски

1. Стабильный изотоп — это изотоп, который не подвергается радиоактивному распаду (ядерному распаду).
Нестабильный изотоп — это изотоп, который действительно подвергается радиоактивному распаду, и поэтому количество встречающегося в природе нестабильного изотопа со временем будет уменьшаться….
Изотопные содержания могут изменяться со временем, если радиоактивный изотоп распадается с образованием стабильного изотопа другого элемента, то изотопное содержание этого стабильного изотопа со временем будет увеличиваться.
Искусственные ядерные реакции также изменят изотопное содержание.
В учебнике по датированию углерода-14 обсуждается изменение содержания изотопов.

2. Масса протона = 1,673 × 10 -27 кг = 1,01 ед.
Масса нейтрона = 1.675 × 10 -27 кг = 1,01 ед.
Масса электрона = 9,109 × 10 -31 кг = 0,000549 ед.

3. Фактически мы оцениваем массу ядра, а не атома (поскольку мы игнорируем массу электронов, которые вносят очень небольшой вклад в массу атома).
Когда мы измеряем массу ядра, мы обнаруживаем, что наблюдаемая масса меньше суммы масс всех нуклонов. Это называется дефектом массы.

Данные по изотопному содержанию

руб.
Элемент Изотоп Обилие (%)
водород 1 H 99.99
2 H 0,01
гелий 3 He 0,0001
4 He 99,9999
литий 6 Li 7,42
7 Li 92.58
бериллий 9 Be 100,00
бор 10 В 19,80
11 B 80,20
углерод 12 С 98.90
13 С 1,10
азот 14 99,63
15 N 0,37
кислород 16 O 99,76
17 O 0.038
18 O 0,20
фтор 19 ф 100,00
неон 20 Ne 90,60
21 Ne 0.26
22 Ne 9,20
натрий 23 Na 100
магний 24 мг 78,90
25 мг 10.00
26 мг 11,10
алюминий 27 Al 100
кремний 28 Si 92,23
29 Si 4,67
30 Si 3.10
фосфор 31 п 100
сера 32 S 95,02
33 S 0,75
34 S 4,21
36 S 0.02
хлор 35 Класс 75,77
37 Класс 24,23
аргон 36 Ар 0,34
38 Ar 0,063
40 Ar 99.60
калий 39 К 93,20
40 К 0,012
41 К 6,73
кальций 40 Ca 96,95
42 Ca 0.65
43 Ca 0,14
44 Ca 2,086
46 Ca 0,004
48 Ca 0,19
скандий 45 SC 100
титан 46 Ti 8.00
47 Ti 7,30
48 Ti 73,80
49 Ti 5,50
50 Ti 5,40
ванадий 50 В 0,25
51 В 99.75
хром 50 Cr 4,35
52 Cr 83,79
53 Cr 9,50
54 Cr 2,36
марганец 55 Мн 100
утюг 54 Fe 5.80
56 Fe 91,72
57 Fe 2,20
58 Fe 0,28
кобальт 59 Со 100
никель 58 Ni 68.27
60 Ni 26,10
61 Ni 1,13
62 Ni 3,59
64 Ni 0,91
медь 63 Cu 69,17
65 Cu 30.83
цинк 64 Zn 48,60
66 Zn 27,90
67 Zn 4,10
68 Zn 18,80
70 Zn 0,60
галлий 69 Ga 60.10
71 Ga 39,90
германий 70 Ge 20,50
72 Ge 27,40
73 Ge 7,80
74 Ge 36,50
76 Ge 7.80
мышьяк 75 Как 100
селен 74 SE 0,90
76 SE 9,00
77 SE 7,60
78 SE 23.50
80 SE 49,60
82 SE 9,40
бром 79 Br 50,69
81 Br 49,31
криптон 78 Kr 0.35
80 Kr 2,25
82 Kr 11,60
83 Kr 11,50
84 Kr 57,00
86 Kr 17,30
рубидий 85 72.17
87 руб. 27,84
стронций 84 Sr 0,56
86 Sr 9,86
87 Sr 7,00
88 Sr 82,58
иттрий 89 Y 100
цирконий 90 Zr 51.45
91 Zr 11,27
92 Zr 17,17
94 Zr 17,33
96 Zr 2,78
ниобий 93 Nb 100
молибден 92 Пн 14.84
94 Пн 9,25
95 Пн 15,92
96 Пн 16,68
97 Пн 9,55
98 Пн 24,13
100 Пн 9.63
рутений 96 Ру 5,52
98 Ру 1,88
99 Ру 12,70
100 Ру 12,60
101 Ру 17,00
102 Ру 31.60
104 Ру 18,70
родий 103 Rh 100
палладий 102 Pd 1,02
104 Pd 11,14
105 Pd 22.33
106 Pd 27,33
108 Pd 24,46
110 Pd 11,72
серебристый 107 Ag 51,84
109 Ag 48,16
кадмий 106 КД 1.25
108 КД 0,89
110 КД 12,49
111 КД 12,80
112 КД 24,13
113 КД 12,22
114 КД 28.73
116 КД 7,49
индий 113 В 4,30
115 дюйм 95,70
банка 112 Sn 0,97
114 Sn 0.65
115 Sn 0,36
116 Sn 14,70
117 Sn 7,70
118 Sn 24,30
119 Sn 8,60
120 Sn 32.40
122 Sn 4,60
124 Sn 5,60
сурьма 121 Сб 57,30
123 Сб 42,70
теллур 120 Te 0.096
122 Te 2,60
123 Te 0,91
124 Te 4,82
125 Te 7,14
126 Te 18,95
128 Te 31.69
130 Te 33,80
йод 127 I 100
ксенон 124 Xe 0,10
126 Xe 0,09
128 Xe 1.91
129 Xe 26,40
130 Xe 4,10
131 Xe 21,20
132 Xe 26,90
134 Xe 10,40
136 Xe 8.90
цезий 133 CS 100
барий 130 Ba 0,11
132 Ba 0,10
134 Ba 2,42
135 Ba 6.59
136 Ba 7,85
137 Ba 11,23
138 Ba 71,70
лантан 138 La 0,09
139 La 99,91
церий 136 CE 0.19
138 CE 0,25
140 CE 88,48
142 CE 11,08
празеодим 141 Pr 100
неодим 142 Nd 27.13
143 Nd 12,18
144 Nd 23,80
145 Nd 8,30
146 Nd 17,19
148 Nd 5,76
150 Nd 5.64
самарий 144 См 3,10
147 Sm 15,00
148 Sm 11.30
149 Sm 13,80
150 Sm 7,40
152 Sm 26.70
154 Sm 22,70
европий 151 Eu 47,80
153 Eu 52,20
гадолиний 152 Gd 0,20
154 Gd 2.18
155 Gd 14,80
156 Gd 20,47
157 Gd 15,65
158 Gd 24,84
160 Gd 21,86
тербий 159 Tb 100
диспрозий 156 Dy 0.06
158 Dy 0,10
160 Dy 2,34
161 Dy 18,90
162 Dy 25,50
163 Dy 24,90
164 Dy 28.20
гольмий 165 Ho 100
эрбий 162 Er 0,14
164 Er 1,61
166 Er 33,60
167 Er 22.95
168 Er 26,80
170 Er 14,90
тулий 169 TM 100
иттербий 168 Yb 0,13
170 Yb 3.05
171 Yb 14.30
172 Yb 21,90
173 Yb 16,12
174 Yb 31,80
176 Yb 12,70
лютеций 175 Лю 97.40
176 Лю 2,60
гафний 174 Hf 0,16
176 Hf 5,20
177 Hf 18,60
178 Hf 27,10
179 Hf 13.74
180 Hf 35,20
тантал 180 Ta 0,012
181 Ta 99,99
вольфрам 180 Вт 0,013
182 Вт 26.30
183 Вт 14.30
184 Вт 30,67
186 Вт 28,60
рений 185 Re 37,40
187 Re 62,60
осмий 184 Ос 0.02
186 Ос 1,58
187 Ос 1,60
188 Ос 13,30
189 Ос 16,10
190 Ос 26,40
192 Ос 41.00
иридий 191 Ir 37,30
193 Ir 62,70
платина 190 Pt 0,01
192 Pt 0,79
194 Pt 32.90
195 Pt 33,80
196 Pt 25,30
198 Pt 7,20
золото 197 Au 100
ртуть 196 Hg 0.15
198 Hg 10,10
199 Hg 17,00
200 Hg 23,10
201 Hg 13,20
202 Hg 29,65
204 Hg 6.80
таллий 203 Tl 29,52
205 Tl 70,48
свинец 204 Пб 1,40
206 Pb 24,10
207 Pb 22.10
208 Pb 52,40
Справка по атомной массе

| Руководство по стехиометрии

Атомная масса

Наша масса — атомная

Помните химические структуры и формулы? Да, они все еще важны. Мы собираемся использовать их для изучения взаимосвязи между массой атомов и массой молекул. Мы не говорим об отношениях типа «это сложно». Это больше похоже на отношения между членами группы друзей, а друзья — это разные типы элементов.Эти отношения помогут нам изучить состав соединений и то, как этот состав может измениться во время реакции. Когда мы закончим, стехиометрия будет казаться легкой, как лимонный сок.

Масса атома определяется количеством протонов, нейтронов и электронов, которые он содержит. Но как определить вес или массу атома? Не то чтобы у нас были достаточно маленькие весы, чтобы измерить такой небольшой вес. (Хотя мы слышали, что Смурфы прячут одного.) Но это нас не остановит.Мы можем определить массу одного атома относительно массы другого атома.

Первым шагом является присвоение значения массы выбранному элементу, который будет действовать как стандарт. Удачный элемент — углерод. Одна единица атомной массы (а.е.м.) определяется как масса, равная одной двенадцатой массы одного атома углерода-12. Кто это придумывает? Атом углерода-12 — это атом, который имеет шесть протонов и шесть нейтронов. Атомная масса определяется как масса атома, выраженная в атомных единицах массы.

Как эта информация может помочь нам определить атомные массы других элементов? Можно провести эксперименты, чтобы определить относительную массу элемента по сравнению с углеродом. Например, атом водорода на 8,4% массивнее атома углерода-12. Это означает, что атомная масса атома водорода составляет 8,4% от массы атома углерода-12 (1,008 а.е.м., если быть точным).

Все еще не уверены? Подумайте об этом так. Скажем, все весы в мире сломаны. Какая жалость. Чтобы исправить это затруднительное положение, мы могли бы создать новую систему весов, которая сравнивает наш вес с другими.Допустим, мы все сравниваем себя с Халком. Мы скажем, что Халк весит ровно одну единицу Халка (HU). Если экспериментально установлено, что ваш вес составляет примерно 22% от веса Халка, вы будете весить 0,22 HU.

Другими словами, нет необходимости помещать крошечный атом на весы, чтобы взвесить его, если у нас есть эталон, с которым мы можем его сравнить. Хотя мы не знаем, какова средняя масса атома железа, мы с по знаем, что он в 56 раз массивнее атома водорода. Вот откуда берутся все атомные массы в периодической таблице.Принесите это на свой следующий званый обед.

Таблица Менделеева: атомные массы расположены под символами элементов.

Возможно, вы обычный Шерлок Холмс, и вы заметили, что атомная масса, указанная для углерода (C) в таблице Менделеева выше, составляет 12,01 а.е.м. Подожди, Шмооп, это , а не 12.00 а.е.м, как мы определили ранее. Что дает? Не бойся, сыщик — этому есть разумное объяснение. Разница в том, что большинство элементов (включая углерод) имеют более одного изотопа .Изотопы — это атомы одного и того же элемента, которые содержат разное количество нейтронов в своих ядрах. Таблица Менделеева показывает средней массы смеси изотопов для каждого элемента.

Относительное содержание изотопов углерода.

Углерод, например, имеет два встречающихся в природе изотопа, углерод-12 и углерод-13. 1 Мы говорим о естественном происхождении, потому что другие изотопы углерода могут быть получены в лаборатории. Пока мы это делаем, вы также можете увидеть углерод-12, записанный как 12 C, или углерод-13, записанный как 13 C.Не волнуйся. Оба обозначения верны. Число в верхнем индексе или после тире указывает на массовое число ( A ) атома, которое представляет собой количество протонов и нейтронов. Не путать с атомным номером ( Z ), который является числом протонов.

https://media1.shmoop.com/images/chemistry/chembook_stoich_graphik_6.png
Протоны, нейтроны и числа, о боже.

Мы можем вычислить среднюю атомную массу углерода, используя вес каждого изотопа и процентное естественное содержание каждого изотопа.

Проверьте эти данные для изотопов углерода 1 :

Изотоп Процентное содержание (%) Атомная масса (а.е.м.)
углерод-12,90 12.00
carbon-13 1,10 13.00335


Среднюю атомную массу можно рассчитать, умножив процентное содержание на атомную массу для каждого изотопа, а затем сложив значения.Проверьте это:

Средняя атомная масса углерода = (процентное содержание 12 C) (атомная масса 12 C) + (процентное содержание 13 C) (атомная масса 13 C)

Средняя атомная масса углерода = (0,9890) (12,000 а.е.м.) + (0,0110) (12,00335 а.е.м.)
= 12,01 а.е.м.

Не забудьте преобразовать проценты в доли перед выполнением вычислений, иначе вы получите возмутительно неверную атомную массу. Например, 97,63 процента превращается в 97.63/100 или 0,9763.

Есть несколько способов проверить ответы и избежать ошибок в вопросах, связанных с вычислением средней атомной массы. Поскольку в природе существует намного больше атомов углерода-12, чем атомов углерода-13 (98,90% против 1,10%), можно ожидать, что средняя атомная масса ближе к 12 а.е.м., чем к 13 а.е.м. Уф, пока все хорошо. Кроме того, у вас есть встроенный ключ для ответа в виде таблицы Менделеева. Всегда перепроверяйте свои ответы с ответами под символами элементов.

Другая распространенная проблема, которая может возникнуть во время викторины или экзамена, — это вычисление атомной массы конкретного изотопа с учетом его содержания, массы и процентного содержания других изотопов.Вас также могут попросить вычислить среднюю атомную массу. Используя то же уравнение, которое мы использовали ранее, мы решим другой параметр.

Ознакомьтесь с некоторыми данными по азоту (N) 1 :

Изотоп Процентное содержание (%) Атомная масса (а.е.м.)
азот-14 99,632 14.003535
азот-15 0,368 ?

Можете ли вы определить атомную массу азота-15? Будем ждать.

Средняя атомная масса азота = (процентное содержание 14 N) (атомная масса 14 N) + (процентное содержание 15 N) (атомная масса 15 N)

Введите то, что вы знаете, и решать остальное. Помните, что среднюю атомную массу азота можно получить из таблицы Менделеева. Это значение составляет 14,0067 а.е.м.

14,0067 а.е.м. = (0,99632) (14,0031 а.е.м.) + (0,00377) (атомная масса 15 N)

Делайте свое дело, и вы получите:

Атомная масса 15 N = 15.0001 amu

Brain Snack

Слово «стехиометрия» происходит от двух греческих слов: stoicheion (значение элемента) и metron (значение меры). 2

Как найти относительную массу

Относительная масса — важное понятие в химии. Он существует, чтобы упростить процесс определения массы атома или молекулы. В абсолютных единицах массы протонов и нейтронов составляют порядка 10 27 килограммов, что составляет одну миллиардную миллиардную часть миллиардной доли килограмма, а масса электронов еще меньше — около 10 30 килограммов, примерно в тысячу раз меньше, чем протон или нейтрон.С этим было бы трудно справиться в практических ситуациях, поэтому ученые определяют относительную атомную массу атома углерода как 12 и рассчитывают все остальное на этой основе.

TL; DR (слишком долго; не читал)

Найдите относительную массу любого атома, прибавив количество протонов к количеству нейтронов. Водород имеет относительную атомную массу 1, а углерод-12 имеет относительную атомную массу 12.

Изотопы одного и того же элемента имеют разное количество нейтронов, поэтому вам нужно рассчитывать для одного конкретного изотопа.Периодические таблицы показывают относительную атомную массу как нижнее число для элемента, но при этом учитываются любые изотопы.

Найдите относительные молекулярные массы, сложив вклад каждого элемента. Используйте химическую формулу, чтобы найти, сколько атомов каждого из них входит, умножьте их относительные атомные массы на количество атомов каждого присутствующего, а затем сложите их все, чтобы найти результат.

Что такое относительная масса?

Относительная масса — это масса атома или молекулы относительно массы 1/12 атома углерода-12.Согласно этой схеме нейтральный атом водорода имеет массу 1. Вы можете думать об этом, как если бы каждый протон или нейтрон считали за 1 и игнорировали массы электронов, потому что они настолько малы по сравнению с ними. Таким образом, формула для относительной атомной массы проста:

Относительная атомная масса = количество протонов + количество нейтронов

Однако, поскольку ученые установили атом углерода-12 в качестве «стандартного атома», техническое определение таково:

Относительная атомная масса = масса атома ÷ (1/12 массы атома углерода-12)

Относительная атомная масса элемента

Элементы являются основными строительными блоками атомов, созданными в результате Большого взрыва или в звезды, и они представлены в периодической таблице.Относительная атомная масса — это нижнее число в периодической таблице (верхнее число — это атомный номер, который подсчитывает количество протонов). Вы можете прочитать это число прямо из упрощенных периодических таблиц для многих элементов.

Однако технически точные периодические таблицы учитывают существование различных изотопов, и указанные в них относительные атомные массы не являются целыми числами. Изотопы — это версии одного и того же элемента с разным количеством нейтронов.

Вы всегда можете найти относительную массу элемента, добавив количество протонов к количеству нейтронов для конкретного изотопа рассматриваемого элемента.Например, атом углерода-12 имеет 6 протонов и 6 нейтронов, поэтому его относительная атомная масса равна 12. Обратите внимание, что когда указан изотоп атома, число после названия элемента является относительной атомной массой. Итак, уран-238 имеет относительную массу 238.

Периодическая таблица и изотопы

Относительные атомные массы в периодической таблице включают вклад различных изотопов путем получения средневзвешенного значения масс различных изотопов на основе их содержания.Хлор, например, имеет два изотопа: хлор-35 и хлор-37. Три четверти хлора, встречающегося в природе, — это хлор-35, а оставшаяся четверть — хлор-37. Формула, используемая для относительных масс в периодической таблице:

Относительная атомная масса = (масса изотопа 1 × содержание изотопа 1 + масса изотопа 2 × содержание изотопа 2 +…) ÷ 100

Итак, для хлора это :

Относительная атомная масса = (35 × 75 + 37 × 25) ÷ 100

= (2,625 + 925) ÷ 100 = 35.5

Для хлора относительная атомная масса в периодической таблице показывает 35,5 в соответствии с этим вычислением.

Относительная молекулярная масса

Просто сложите относительные массы составляющих элементов, чтобы найти относительную массу молекулы. Это легко сделать, если вы знаете относительные атомные массы рассматриваемых элементов. Например, вода имеет химическую формулу H 2 O, поэтому есть два атома водорода и один атом кислорода.

Рассчитайте относительную молекулярную массу, умножив относительную атомную массу каждого атома на количество этих атомов в молекуле, а затем сложив результаты.Это выглядит так:

Относительная молекулярная масса = (количество атомов элемента 1 × относительная масса элемента 1) + (количество атомов элемента 2 × относительная масса элемента 2) +…

Для H 2 O, элемент 1 — водород с относительной атомной массой 1, а элемент 2 — кислород с относительной атомной массой 16, поэтому:

Относительная молекулярная масса = (2 × 1) + (1 × 16) = 2 + 16 = 18

Для H 2 SO 4 элемент 1 — водород (H), элемент 2 — сера (S с относительной массой = 32), а элемент 3 — кислород (O), поэтому тот же расчет дает:

Относительная молекулярная масса H 2 SO 4 = (количество атомов H × относительная масса H) + (количество атомов S × относительная масса S) + (количество атомов O × относительная масса of O)

= (2 × 1) + (1 × 32) + (4 × 16)

Вы можете использовать тот же подход для любой молекулы.

масс

масс

Точные массы и соотношения изотопов
Элемент Символ Номинальная масса Точная масса Изобилие X + 1 Фактор
*
X + 2 Фактор
*
Водород H
D или 2 H
1
2
1.00783
2,01410
99,99
0,01
Углерод 12 С
13 С
12
13
12.0000
13.0034
98,91
1,09

1.1н С

0,006n С 2
Азот 14 N
15 N
14
15
14.0031
15,0001
99,6
0,37

0,37n N
Кислород 16 O
17 O
18 O
16
17
18
15,9949
16,9991
17,9992
99,76
0,037
0,20

0,04n O


0.2н О
фтор Ф 19 18.9984 100
Кремний 28 Si
29 Si
30 Si
28
29
30
27,9769
28,9765
29,9738
92.28
4,70
3,02

5.1n Si


3.3n Si
фосфор п. 31 30,9738 100
сера 32 S
33 S
34 S
32
33
34
31.9721
32,9715
33,9679
95,02
0,74
4,22

0,78n S


4.4n S
Хлор 35 Класс
37 Класс
35
37
34,9689
36,9659
75,77
24,23

32.5n Класс

Бром 79 руб.
81 руб.
79
81
78.9183
80.9163
50,5
49,5

98.0н Br

Йод I 127 126.9045 100

* X представляет относительную интенсивность иона с наименьшей массой. в кластере изотопных ионов.Для этого расчета установлено значение 100%. Коэффициент умножается на количество атомов (n) указанного элемента, чтобы вычислить вклад интенсивности от изотопов большей массы.

Органическая химия
Государственный университет Мичигана



Эта таблица адаптирована из Introduction to Mass Spectrometry , J.T. Watson

Калькулятор массы справа можно использовать для расчета точной массы молекулы на основе ее элементного состава.Просто введите соответствующий номер нижнего индекса справа от каждого символа, оставив эти элементы пустыми, и нажмите кнопку « Вычислить ». Для этих расчетов используется только масса наиболее распространенного изотопа относительно C (12,0000). Для соединений хлора и брома необходимо добавить 1,997 и 1,998 соответственно для каждого галогена, чтобы получить более высокие значения массовых изотопов.


Калькулятор слева можно использовать для расчета изотопных вкладов в содержание ионов на 1 и 2 а.е.м. больше, чем молекулярный ион (M).Просто введите соответствующий номер нижнего индекса справа от каждого символа, оставив эти элементы пустыми, и нажмите кнопку « Вычислить ». Числа, отображаемые в полях M + 1 и M + 2, относятся к M, установленному на 100%. Конечно, соединения хлора и брома имеют очень большое количество изотопов.
Более элегантный и полный калькулятор структуры изотопов был создан Джунхау Яном. Чтобы использовать этот ресурс, щелкните здесь.
Полезный ресурс, написанный Джефом Розенски, вычисляет все возможные комбинации H, C, N и O, которые дают определенную номинальную массу.Чтобы использовать этот калькулятор, щелкните здесь.


Рассчитайте средний атомный вес по весу и содержанию изотопов

ChemTeam: рассчитайте средний атомный вес на основе веса и содержания изотопов.

Рассчитайте средний атомный вес с учетом веса и содержания изотопов


Пятнадцать примеров

Вернуться к Mole Содержание

Рассчитайте содержание изотопов с учетом атомного веса и веса изотопов.


Для решения этих задач вам понадобится некоторая информация: точный атомный вес для каждого встречающегося в природе стабильного изотопа и его процентное содержание.Эти значения можно найти в стандартном справочнике, таком как «Справочник по химии и физике». Единица, связанная с ответом, может быть а.е.м. или г / моль, в зависимости от контекста вопроса. Если из контекста не ясно, что г / моль является желаемым ответом, используйте аму (что означает атомную единицу массы).

Эту проблему также можно обратить вспять, например, вычислить изотопные содержания, когда заданы атомный вес и веса изотопов. Изучите руководство ниже, а затем посмотрите на проблемы, решенные в обратном направлении.


Пример №1: Углерод

массовое число точный вес процентное содержание
12 12.000000 98.90
13 13.003355 1,10

Чтобы вычислить средний атомный вес, каждый точный атомный вес умножается на его процентное содержание (выраженное в виде десятичной дроби). Затем сложите результаты и округлите до соответствующего числа значащих цифр.

Это решение для углерода:

(12,000000) (0,9890) + (13,003355) (0,0110) = 12,011 а.е.м.

Пример № 2: Азот

массовое число точный вес процентное содержание
14 14.003074 99.63
15 15,000 108 0,37

Это раствор для азота:

(14.003074) (0,9963) + (15,000108) (0,0037) = 14,007 а.е.м.

Видео: как рассчитать средний атомный вес.


23
Пример № 3: Хлор Пример № 4: Кремний
массовое число точный вес процентное содержание массовое число точный вес процентное содержание
35 34. 75,77 28 27.976927 92.23
37 36.
24,23 29 28.976495 4.67
30 29,973770 3,10
Ответ для хлора: 35,453 Ответ для кремния: 28,086

Этот тип расчета может быть выполнен в обратном порядке, когда изотопные содержания могут быть рассчитаны, зная средний атомный вес.Перейти к руководству по обратному направлению.


Пример 5: В образце из 400 атомов лития было обнаружено, что 30 атомов представляют собой литий-6 (6,015 г / моль) и 370 атомов представляют собой литий-7 (7,016 г / моль). Рассчитайте среднюю атомную массу лития.

Решение:

1) Рассчитайте процентное содержание каждого изотопа:

Li-6: 30/400 = 0,075
Ли-7: 370/400 = 0,925

2) Рассчитайте средний атомный вес:

х = (6.015) (0,075) + (7,016) (0,925)

x = 6,94 г / моль


Пример № 6: Образец элемента X содержит 100 атомов с массой 12,00 и 10 атомов с массой 14,00. Рассчитайте среднюю атомную массу (в а.е.м.) элемента X.

Решение:

1) Рассчитайте процентное содержание каждого изотопа:

X-12: 100/110 = 0,909
X-14: 10/110 = 0,091

2) Рассчитайте средний атомный вес:

х = (12.00) (0,909) + (14,00) (0,091)

x = 12,18 а.е.м. (до четырех сигнатур)

3) Вот еще способ:

100 атомов с массой 12 = полная масса атома 1200

10 атомов с массой 14 = полная масса атома 140

1200 + 140 = 1340 (общая масса всех атомов)

Общее количество атомов = 100 + 10 = 110

1340/110 = 12,18 а.е.м.

Первый способ — это стандартный метод решения проблем такого типа. Это потому, что мы обычно не знаем конкретное количество атомов в данном образце.Чаще всего нам известно процентное содержание, которое отличается от конкретного количества атомов в образце.


Пример № 7: Бор имеет атомную массу 10,81 а.е.м. в соответствии с периодической таблицей. Однако ни один атом бора не имеет массы 10,81 а.е.м. Как можно объяснить эту разницу?

Решение:

10,81 а.е.м. — это среднее, а именно средневзвешенное значение. Оказывается, существует два стабильных изотопа бора: бор-10 и бор-11.

Ни один из изотопов не весит 10,81 а.е.м., но вы можете получить 10,81 а.е.м. следующим образом:
х = (10,013) (0,199) + (11,009) (0,801)

х = 1,99 + 8,82 = 10,81


Пример № 8: Медь в природе встречается в виде Cu-63 и Cu-65. Какой изотоп более распространен?

Решение:

Найдите атомный вес меди: 63,546 а.е.м.

Поскольку наше среднее значение ближе к 63, чем к 65, мы пришли к выводу, что Cu-63 является более распространенным изотопом.


Пример № 9: Медь имеет два изотопа природного происхождения. Cu-63 имеет атомную массу 62,9296 а.е.м. и содержание 69,15%. Какова атомная масса второго изотопа? Каков его ядерный символ?

Решение:

1) Найдите атомный вес меди:

63,546 а.е.м.

2) Установите следующее и решите:

(62,9296) (0,6915) + (х) (0,3085) = 63,546

43,5158 + 0,3085х = 63.546

0,3085x = 20,0302

x = 64,9277 а.е.м.

3) Ядерный символ:

2965Cu

4) Вы могли это увидеть

29-Cu-65

Используется в таких ситуациях, как Интернет, где нельзя воспроизвести подстрочные / надстрочные обозначения. Вы также можете увидеть это:

65/29 Cu


Пример № 10: Встречающийся в природе йод имеет атомную массу 126,9045. Образец йода весом 12,3849 г случайно загрязнен 1.0007 г I-129, синтетического радиоизотопа йода, используемого при лечении некоторых заболеваний щитовидной железы. Масса И-129 128,9050 а.е.м. Найдите кажущуюся «атомную массу» загрязненного йода.

Решение:

1) Рассчитайте массу загрязненного образца:

12,3849 г + 1.0007 г = 13,3856 г

2) Рассчитайте процентное содержание (а) природного йода и (б) I-129 в загрязненной пробе:

(а) 12,3849 г / 13,3856 г = 0.


(б) 1.0007 г / 13,3856 г = 0,07476

3) Рассчитайте «атомную массу» загрязненного образца:

(126,9045) (0,

) + (128,9050) (0,07476) = х

x = 127,0540 а.е.м.


Пример № 11: Неон имеет два основных изотопа, Неон-20 и Неон-22. Из каждых 250 атомов неона 225 будут Неон-20 (19,992 г / моль), а 25 — Неон-22 (21,991 г / моль). Какова средняя атомная масса неона?

Решение:

1) Определите процентное содержание (но оставьте в виде десятичной дроби):

Ne-20 —> 225/250 = 0.90
Ne-22 —> 25/250 = 0,10

Последнее значение также можно вычислить вычитанием, в этом случае 1 — 0,9 = 0,1

2) Рассчитайте средний атомный вес:

(19,992) (0,90) + (21,991) (0,10) = 20,19

Пример № 12: Рассчитайте средний атомный вес магния:

массовое число точный вес процентное содержание
24 23.985042 78.99
25 24,985837 10,00
26 25.98 2593 11.01

Ответ? Найдите магний в периодической таблице:


Помните, что приведенный выше метод вычисления среднего атомного веса элемента. Ни один отдельный атом элемента не имеет заданного атомного веса, потому что атомный вес элемента является средним, в частности, называемым «средневзвешенным».Учитывая это, вам могут задать вопрос.

Пример № 13: Серебро имеет атомную массу 107,868 а.е.м. Имеет ли масса атома какого-либо изотопа серебра 107,868 а.е.м.? Объясните, почему да или почему нет.

Решение:

Конкретный вопрос о серебре, но это может быть любой элемент. Ответ, конечно, нет. Атомный вес серебра является средневзвешенным. Серебро не состоит из атомов, каждый из которых весит 107,868.

Пример № 14: Учитывая, что средняя атомная масса водорода в природе равна 1.0079, что это говорит вам о процентном составе H-1 и H-2 в природе?

Решение:

Это говорит о том, что доля H-1 намного больше, чем доля H-2 в природе.

Пример № 15: Относительная атомная масса неона равна 20,18. Он состоит из трех изотопов с массами 20, 21 и 22. Он состоит на 90,5% из Ne-20. Определите процентное содержание двух других изотопов.

Решение:

1) Пусть y% будет относительным содержанием Ne-21.

Добавить комментарий

Ваш адрес email не будет опубликован. Обязательные поля помечены *